Series 66; part 4

Lakukan tugas rumah & ujian kamu dengan baik sekarang menggunakan Quizwiz!

There are many sources of taxable income to an individual. Included might be money received from which of the following? Sole proprietorship Subchapter S corporation Investments Life insurance death benefit A) I, II, and III B) II and III C) I, II, III, and IV D) I and II

A Explanation An individual can generate income from running a sole proprietorship or being a shareholder in an S corporation (the exam will possibly use the obsolete term, Subchapter S). Of course, taxable income can be generated by investments in the form of dividends, interest and capital gains. The assumption here must be that the death benefits are from a life insurance policy because those, unlike the death benefit from an annuity, are not subject to income tax.

Becky Biggins has an executive position with a large corporation that covers her under its defined benefit pension plan. This year, Becky's salary will top $435,000. Becky has no dependents and wishes to maximize funds that she can accumulate for her retirement. Becky could not open a traditional IRA open a traditional IRA but would not be able to deduct her contributions open a Roth IRA not open a Roth IRA A) II and IV B) I and IV C) I and III D) II and III

A Explanation Anyone with earned income can open a traditional IRA. Deductibility of contributions may be disallowed if the individual is covered under a corporate plan and has earnings in excess of a certain level. Becky's salary exceeds the maximum permitted for a single person so her contributions would be made with after-tax dollars. In the case of a Roth, nothing is deductible, so it doesn't matter if you are covered at work. However, Becky's salary is far in excess of the maximum permitted for a single person to contribute to a Roth IRA.

One of your clients invested $10,000 into a mutual fund. The client elected to reinvest all dividends. As a consequence of this, A) the investor's basis is increased by the amount of the reinvested dividends B) taxes are deferred until those shares are redeemed C) the dividends will be taxed as capital gains once the shares are liquidated D) the reinvestments will purchase shares at a discount from the NAV

A Explanation Because the reported dividends are taxed each year, when the shares are ultimately liquidated, they have already been taxed. So, the investors cost basis is increased by the amount of the reinvestment.

Many parents prefer to use a Section 529 plan over a Coverdell ESA to finance their child's education plans because contribution limits are higher funds may be withdrawn tax-free if used for qualified education expenses there are no earnings limits 529 contributions are tax deductible on the federal level A) I and III B) I, II, and III C) I and II D) III and IV

A Explanation Contributions to a Coverdell ESA are limited to $2,000 per beneficiary per year while those to a Section 529 plan can be as high as $300,000 in some states. A married couple cannot make a Coverdell contribution if their income exceeds $220,000, while there is no earnings limit to contribute to a 529. In neither case is the contribution tax deductible on the federal level (although the Section 529 plans may have tax advantages in some states). We are often asked about choice II. The question is asking about differences between the two plans and choice II is true for both of them.

An investment adviser who believes that we are in the early recovery portion of the business cycle would most likely recommend A) cyclical stocks. B) value stocks. C) long-term bonds. D) defensive stocks.

A Explanation Cyclical stocks have a high correlation to the swings in the economy as reflected in the business cycle. The ideal time to purchase stocks that are affected by economic cycles is right as the recovery begins from a trough (or recession). Defensive stocks are good to hold when the economy is in the early contraction period of the cycle. During a recovery, interest rates usually rise and that is not good for holders of long-term bonds because as interest rates go up, bond prices fall. Value stocks might also perform well, but many studies have shown that they offer better relative performance in periods of contraction rather than expansion.

A firm declares a $3.00 cash dividend to its shareholders. The firm has issued dividends of only $.07 per share for each of the last 15 quarters, and market analysts had anticipated a similar dividend this quarter. In an efficient market, one would expect A) a price change upon the announcement. B) a price decrease after the announcement. C) a price increase before the announcement. D) no price change before or after the announcement.

A Explanation In an efficient market, the price of the stock will represent all information that is public. Because the increase in the dividend was not public knowledge until it was declared, no price change would take place before the announcement. A price change, representing the increase in dividends, would be expected immediately after the information became public.

An individual covered under a traditional IRA would not be able to also maintain which of the following? A) 401(k) and 403(b) B) 403(b) and 457 C) Roth IRA D) 401(k) and 457

A Explanation One cannot have a 401(k) and a 403(b) at the same time. One of the features of a 457 plan is that the participant can also have other retirement plans such as a 401(k) or a 403(b). Remember, anyone with earned income can have an IRA, traditional, Roth, or both.

If an investment adviser's client wishes to save current income taxes by placing certain investments in a charitable trust, ethically, the investment adviser should A) recommend the client consult with a qualified attorney B) refuse to discuss the trust with the client because the adviser is not an attorney C) help the client draft the appropriate documents following a discussion of the advantages of the arrangement D) urge the client to consult with an attorney who pays a referral fee to the investment adviser

A Explanation Presuming the adviser is not a licensed attorney, he should recommend the client see a qualified attorney. However, it is ethical to discuss the nature of a charitable trust with the client.

The Employment Retirement Income Security Act of 1974 (ERISA) is A) a federal law regulating many aspects of private retirement plans B) a state law establishing a pension system for state employees C) a state law regulating many aspects of private retirement plans D) a federal law establishing the Social Security system

A Explanation The Employment Retirement Income Security Act of 1974 (ERISA) is a federal law that regulates many aspects of retirement plans established by employers in the private sector. It was enacted to help protect the rights of employees who participate in retirement plans. ERISA provides many of the rules that retirement plans must meet if they want to be considered qualified plans.

If an investor buys a utility stock with a stable 5% dividend, and after a year the investor's total return in the stock is 10%, the most likely reason for this is A) the stock appreciated by 5% B) the company doubled its dividend payment C) the stock price declined D) the investor reinvested the quarterly dividends

A Explanation The most likely cause for the total return was an increase in the stock price.

When saving money for a child's college education, one consideration is the impact that those savings will have on the child's eligibility for financial aid. Funds saved in which of the following vehicles has the most detrimental effect on financial aid? A) Coverdell ESA B) UTMA C) Section 529 D) Prepaid tuition plan

B Explanation Assets held in custodial accounts (UTMA or UGMA) are counted at 20% of their value, which compares unfavorably with the 5.64% valuation of Section 529 or Coverdell ESA assets. Please note: It is highly unlikely that you will need to know the percentages - but you will need to know that custodial accounts do not receive as beneficial treatment when applying for financial aid.

All of the following statements concerning capital market theory are correct EXCEPT A) the market risk premium is the difference between the expected return for the equities market and the risk-free rate of return. B) beta is a measure of volatility, or relative unsystematic risk, for stock or portfolio returns. C) the security market line (SML) depicts the tradeoff between risk and expected return for all assets, whether individual securities, inefficient portfolios, or efficient portfolios. D) the security market line (SML) is the graphical depiction of the capital asset pricing model (CAPM).

B Explanation Beta is a measure of relative systematic risk for stock or portfolio returns. A stock or portfolio with a beta of 1.0 would have the same systematic risk as the overall market.

GEMCO Manufacturing Co. has appointed the company's CFO as the trustee for their employee retirement plan. You are an IAR and you advise a substantial portion of the plan's assets. You are contacted by the CFO requesting a short-term loan from the plan assets for which he will pay the plan prime + 2%. Your best course of action would be to A) permit the loan once you have been satisfied that there is adequate collateralization in place B) refuse to allow this to happen because it would be a violation of your fiduciary responsibility C) refuse to allow this to happen because the plan assets will suffer D) permit the loan because the CFO is the plan trustee

B Explanation ERISA never permits transactions of this type for a plan trustee. As an IAR handling some of the plan's investments, you would be placed in a fiduciary position and could not violate that trust.

A nonqualified, single premium variable annuity differs from a Keogh plan in that A) earnings are tax deferred B) all payouts are fully taxable in a Keogh plan C) it is open to self-employed persons D) both are subject to early withdrawal penalties

B Explanation Earnings on investments made in both a Keogh plan and nonqualified annuity grow on a tax-deferred basis; they are not taxed until withdrawn. The cost basis in a Keogh plan is zero because contributions are tax deductible, but distributions are fully taxable upon receipt. However, in a nonqualified annuity, the cost basis is equal to the amount invested because the contributions are nondeductible; only the earnings portion of the distributions is taxable.

Each of these would be considered an advantage of using a 529 plan rather than a Coverdell ESA to fund a child's future education except A) the 529 plan has no earnings limitation on the donor. B) the 529 plan is counted at a lower percentage of assets when applying for financial aid. C) the 529 plan has no age limits. D) the 529 plan allows for higher contribution levels.

B Explanation Funds in both plans are counted as assets of parents at 5.64% if owner is a parent or dependent student, so there is no difference. The 529 plan allows for far greater contribution levels and there is no income limitation on the donor as exists with the Coverdell ESA. The funds in the ESA must be used by the time the beneficiary is 30; no such age restrictions apply to the 529 plan.

Which of the following statements is TRUE regarding Section 529 plans? Funds withdrawn for qualified education expenses are always free of federal income tax. Funds withdrawn for qualified education expenses are always free of state income tax. The maximum contribution limits are determined on a federal level. The maximum contribution limits are determined on a state level. A) II and III B) I and IV C) I and III D) II and IV

B Explanation Section 529 plan withdrawals are exempt from federal income tax if used for the right expenses. In almost all cases, if the plan is one operated by your state of residence, it will be exempt from your state's income tax. But, if you elect to contribute to a plan operated by another state, more than likely, any withdrawals will be subject to your state's income tax. Because the plans are state operated, the maximum contribution limits are set by each state.

Gaston is a police officer and wishes to contribute to a retirement plan sponsored by the city. Gaston wants the flexibility of being able to have unfettered penalty-free access to his funds before reaching age 59½. This can only be accomplished if Gaston contributes to A) a 401(k) plan. B) a 457 plan. C) a SEP-IRA. D) a 403(b).

B Explanation The 457 plan is unique in that it is the only tax-qualified retirement plan permitting withdrawals, for any reason, before reaching 59½ without penalty. All qualified plans have exceptions to the 10% penalty tax, but only the 457 allows the withdrawals for any reason.

The capital asset pricing model (CAPM) is based on several limiting assumptions. Which of the following statements is correct regarding the CAPM? A) The CAPM does not assume that investors have access to the same information. B) The CAPM assumes that the optimal portfolio should be the one with the highest Sharpe ratio of all possible portfolios. C) The CAPM assumes that investors' expectations regarding risk and return are not identical but normally distributed. D) The CAPM does not assume that the expected excess returns for the market are known.

B Explanation The CAPM assumes that investors should construct a portfolio with the highest Sharpe ratio because that offers the highest risk-adjusted return. It also assumes that the expected excess returns for the market are assumed to be known in that investors have access to the same information. As well, it assumes that returns are normally distributed and investors' expectations for risk and return are identical.

A customer is selling inherited stock. The decedent originally paid $50 per share and on the date of the decedent's death, the stock was worth $60 per share. On the day the customer sells the stock, the price per share is $62. What is the investor's cost basis in the stock? A) 55 B) 60 C) 50 D) 62

B Explanation The IRS allows a step-up in basis for inherited stock. The customer's cost basis is the fair market value of the stock on the date that the decedent died.

Using industry jargon, the tax on the last dollar of income is at A) the effective rate B) the marginal rate C) the average rate D) the final rate

B Explanation The IRS defines marginal tax rate as "the highest rate that you will pay on your income." Basically, as you make more money, you pay tax at a higher rate incrementally. The effective tax rate is the average that you pay on all of your income.

As part of its suitability determination, an IA firm requires that all potential nonbusiness clients complete a family balance sheet. Items that would be included are gold jewelry loan secured by the family automobile the amount paid thus far this year for Botox injections the balance owed to the dentist for new crowns A) I and IV B) I, II and IV C) II and III D) I, II, III and IV

B Explanation The balance sheet contains assets and liabilities as of a specific point in time. Personal property currently owned, such as jewelry, is an asset. A loan still outstanding, such as the car loan and the debt to the dentist, are liabilities. The amount already paid for the Botox injections is no longer on the balance sheet.

The capital asset pricing model (CAPM) is used by many to assess the expected return of a security. If the current risk-free rate is 2%, the current return on the market is 12%, and a particular stock's beta is 0.8 with a correlation coefficient of 0.60, the expected return would be A) 7.2% B) 10.0% C) 11.6% D) 9.6%

B Explanation The formula for this computation is as follows: 12% (the return on the market is a beta of 1.0) minus the risk-free rate of 2%, or 10%. Then, multiply that by the beta of this stock (0.8) to arrive at 8%. That is, the stock should return 8% above the risk-free rate of 2%, or 10%. The correlation coefficient is not relevant to this computation. capital asset pricing model: Return MINUS risk free X BETA + risk free .12 - .02 = .10 X .8 = .08 + .02 = .10

An investor invests a total of $30,000 and creates a portfolio of 3 different stocks placing 1/3 of his investment into each security. From his holding in Company A, he receives a dividend of $600; from Company B, a dividend of $500; and from Company C, no dividend. One year later, the market price of the Company A stock has increased by 20%, Company B's stock increased by 10%; and Company C's stock has remained the same. What is the investor's total return on this portfolio? A) 3.67% B) 13.67% C) 10% D) 4.56%

B Explanation Total return includes both appreciation (growth) and income (dividends). Let's go step by step. Company A's $10,000 original investment increases by 20%, or $2,000 plus dividends of $600. Company B's $10,000 original investment increases by 10%, or $1,000 plus dividends of $500. Company C's $10,000 original investment is unchanged and there is no dividend. Therefore, we have appreciation of $3,000 plus dividend income of $1,100. That is a total return of $4,100 on an investment of $30,000, or 13.67%.

An investor buys a 5% AA-rated corporate bond at par. After 1 year, if his total return on the position is 4%, the most likely explanation for this is A) interest rates decreased causing the bond price to increase B) interest rates increased causing the bond price to decrease C) the investor paid accrued interest when he bought the bond diminishing his first year's return D) the bond rating was downgraded

B Explanation Total return is computed by adding together the income received plus any capital gain or loss. Because the bond is purchased at par, selling the bond at a loss is the only way the investor's total return could be less than the coupon rate. When interest rates go up, bond prices go down.

Who is obligated for the payment of taxes in an UTMA account? A) Donor B) Child C) Custodian D) Parent

B Explanation UTMA and UGMA accounts are custodial accounts. They are for the benefit of the child and bear the child's Social Security number. Although in practice the taxes are usually paid by the parent or legal guardian, they are the responsibility of the beneficial minor (child).

Mr. Berg has been charting DMF stock prices. The stock usually fluctuates between 71 and 86. The stock is currently at 84, and the increasing upside volume makes him believe that a breakout is possible. Which of the following would he most likely enter? A) A sell limit at 88 B) A sell stop at 70 C) A buy stop at 88 D) A buy limit at 85

C Explanation A breakout occurs when a security trades outside an established range. In this case, because Mr. Berg has no position, he would want to purchase only if the stock breaks through the resistance level already established.

Under which of the following circumstances would a premature distribution from a traditional IRA be exempt from the premature distribution penalty? A) When the account is fully funded with nondeductible contributions B) A distribution taken at age 55 if the owner is retired C) When the distribution is paid in equal annual amounts over the owner's life D) A distribution taken to satisfy the terms of a court-ordered property settlement

C Explanation A distribution from an IRA taken in equal annual amounts over the owner's life is not subject to the 10% premature distribution penalty even if started before age 59½. This is one of the exceptions that apply to IRAs. The exception for qualified domestic relations orders (QDROs) and for retirement at age 55 apply to employer-sponsored plans but not to IRAs.

Under ERISA Section 404(c), plan participants must be able to reallocate plan assets A) annually B) daily C) once every 3 months D) once every week

C Explanation Although many 401(k) plans provide for daily reallocation of plan assets, Section 404(c) requires that plan participants be able to reallocate plan assets, through internet trading or other methods, at least once every 3 months.

All of the following are advantages of a 401(k) plan EXCEPT A) employees and the business may reduce current taxes B) the owner of the business may participate in the plan C) the employer may make unlimited contributions, which generate unlimited tax deductions for the business D) tax deferral on the plan earnings is advantageous to employees

C Explanation Contributions are deductible by the employer but are not unlimited because contributions to a 401(k) are subject to a number of limits. Tax deferral on plan earnings is advantageous to employees. The owner of the business may participate in the plan.

Which of the following is NOT required under ERISA Section 404(c)? A) Plan participants must have access to a broad range of investment alternatives. B) Each plan participant must have the ability to exercise independent control over assets in her account. C) All plan participants must have been employed by the plan sponsor for a minimum of 3 years. D) Individual accounts must be provided for each plan participant.

C Explanation ERISA Section 404(c) relieves the employer of fiduciary responsibility for investment decisions made by employees. To qualify for this protection, employees must enjoy the benefits and risks of their decisions (individual accounts), have the right to exercise independent control over the account, and have a sufficiently broad range of choices to make the right of control meaningful. Section 404(c) has nothing to do with the employee's length of employment.

When operating a Keogh plan, a self-employed individual must make contributions for A) part-time employees who have worked for the company for 3 or more years B) all employees scheduled to work for 1,000 hours per year or more C) full-time employees who are at least 21 years old and have worked for the company for 1 or more years D) all employees

C Explanation Employees must be covered under a Keogh plan if they are at least 21 years old, have been employed a minimum of 1 year, and work full time (at least 1,000 hours per year). Keogh plans do not include employees who are under 21 or have just started working with the employer.

In the formula for determining the real rate of return, A) the marginal tax bracket is subtracted from the investment return B) the inflation rate is divided by the investment return C) the inflation rate is subtracted from the investment return D) the investment return is divided by the inflation rate

C Explanation In computing the real rate of return, which represents inflation-adjusted compounding (or discounting), a formula is applied in which the rate of inflation (usually as measured by the CPI) is subtracted from the investor's rate of return.

Suzie McQueen has a very successful interior design shop she has run as a sole proprietorship. She has just celebrated her 60th birthday and has been giving thought to an eventual sale of the business. She wants your opinion on whether she should incorporate or change to a partnership. You might respond that A) the partnership form of business structure would be the easiest for ultimate transfer of ownership B) the corporate form of business structure would be the least expensive to form C) the corporate form of business structure would be the easiest for ultimate transfer of ownership D) the partnership form of business structure would enable Suzie to maximize her sale price

C Explanation In general, the corporate form of business leads to the easiest transfer of ownership. Because Suzie would probably own 100% of the stock, all she would have to do is sell that stock to a new purchaser and the corporation could continue just as before. If Suzie wanted to reorganize as a partnership, she would have to bring in at least one additional individual, ending her total ownership of the business. Even then, a partnership interest is not as easy to sell as stock.

Investors who buy shares in state-specific municipal bond funds may be subject to A) federal income tax B) out-of-state property tax C) capital gains tax D) no taxation

C Explanation Interest received from municipal bonds and municipal bond funds is generally income tax-free on a federal basis, but taxable in states other than the state of issue. State-specific funds avoid that problem. These investments are subject to capital gains taxes if sold at prices above investors' cost.

A new client is opening a margin account and notices the following wording in the documentation: "You are authorized to lend to yourself or others any securities held by you in my margin account and to carry all securities lent as general loans, and you shall have no obligation to retain under your possession and control a like amount of such securities." When the client asks you what this is about, you would respond that A) this is the hypothecation agreement B) if the client does not sign the document, the account cannot be opened C) this is the loan consent agreement D) this is the credit agreement

C Explanation No broker-dealer shall lend securities that are held on margin for a customer and that are eligible to be pledged or loaned, unless the broker-dealer shall first have obtained a written authorization from such customer permitting the lending of such securities. That written authorization is known as the loan consent agreement and is the only one of the margin documents that is optional.

One of your ultra-high net worth clients would like to give some low cost basis stock as gifts to her adult grandchildren. It would be prudent for you to tell her that A) making the gift under the Uniform Transfer to Minors Act is generally the most advantageous for the child. B) for purposes of the gift tax, her cost basis will be used. C) unlike an inheritance, there is no stepped-up cost basis. D) it would be wise for her to use a TOD account to avoid probate.

C Explanation One of the benefits of inheriting low cost basis securities is the stepped-up basis and that does not apply to gifts. Although the donor will not be the one subject to capital gains tax, it would be the right thing to do to let her know that the donees (her grandchildren) will be receiving the stock at her cost basis. TOD would not apply to stock that is the subject of a gift; it is only when the stock remains in the grandmother's name and has been designated for the grandchildren after her death. When computing the value of a gift to determine if there is a gift tax obligation, it is the fair market value of the gift that is used. Finally, the question states these are adult grandchildren - UTMA would not apply to them.

George and Martha Washington are both in their mid-70s, are very active in their community, and both plan to start working part time at the local community bank. They would like to contribute a small portion of their earnings to some form of retirement plan. Which of the following choices would be the most appropriate for this couple? A) A Keogh Plan B) A traditional IRA C) A Roth IRA D) The bank's 401(k) plan

C Explanation One of the distinguishing characteristics of the Roth IRA is that there are no required minimum distributions (RMDs) once the taxpayer attains age 72. At their age, opening a traditional IRA would mean they would be investing into the plan, but withdrawing at the same time. That does not help them accumulate funds for the future. Because they will be employed by the bank, they are not eligible for a Keogh plan. If the bank offers a 401(k) plan, it is unlikely they would be eligible. Although part-time employees who work at least 500 hours per year may be covered, coverage does not start until they've worked for at least three years.

The Sharpe ratio is a measurement of a portfolio's A) inflation-adjusted return B) holding period return C) risk-adjusted return D) after-tax rate of return

C Explanation The Sharpe ratio measures the risk-adjusted return. A higher Sharpe ratio indicates that, based on the risk taken, the investor's return is better than expected.

To assess the performance of a small-cap stock fund you compare its results against A) the S&P 100 B) the S&P 500 C) the Russell 2000 D) the Dow Jones Industrial Average

C Explanation The appropriate benchmark for a small-cap fund is the Russell 2000 because it is composed of similar companies. Little Russell

One measure of an investor's total return is called holding period return. The computation includes both income and appreciation and is used for both debt and equity securities. An investor's holding period return would be less than the bond's yield to maturity if A) the bond was called at a discount B) the bond was redeemed at a premium C) the coupons were reinvested at a rate below the yield to maturity D) the investor purchased a put option on the bond

C Explanation The calculation of yield to maturity assumes reinvestment of the bond's interest at the coupon rate. Therefore, if the investor was only able to do less than that, the holding period return would be decreased. This is part of the concept of internal rate of return (IRR), which takes into consideration the time value of money (compounding). It is tempting to choose the answer "a call at a discount," but bonds are never called at a price below par. Just keep it simple: If the question says you can earn less than the YTM, your return will be lower than the quoted YTM.

If you knew a given stock had a 40% chance of earning a 10% return, a 40% chance of earning −20%, and a 20% chance of earning −10%, the expected return would be equal to A) 14% B) −10% C) −6% D) 10%

C Explanation The expected return is computed by taking the probability of each possible return outcome, multiplying it by the return outcome itself, and then adding them all together. In this case, the math is as follows: (0.4 × 10%) + (0.4 × −20%) + (0.2 × −10%), or +4% − 8% − 2%, which equals -6%. Part of the trick here is catching the probable negative returns and the ridiculous assumption that an investor would consider looking at a stock with this kind of expected return. You can always count on NASAA to surprise you.

An investment adviser representative is evaluating ABC stock to see if it is a good fit for a client's portfolio. Using the security market line (SML), what is the expected return for ABC when the return on the market is 20%, the 91-day Treasury bill is yielding 4%, ABC's beta is 0.70, and the inflation rate, as measured by the CPI, is 3%? A) 14.0% B) 10.2% C) 15.2% D) 11.2%

C Explanation The formula for this computation is as follows: 20% (the return on the market is a beta of 1.0) minus the risk-free rate of 4%, or 16%. Then, multiply that by the beta of this stock (0.7) to arrive at 11.2%. That is, the stock should return 11.2% over the risk-free rate of 4%, or 15.2%. Inflation rate is only important if we are looking for the real (inflation-adjusted) return, not the expected return. Return - Risk free then multiply by BETA + risk free .2-.04=.16 then .16X.7=11.2 + .04 = 15.2

A person providing which of the following services to an ERISA plan would be performing in a fiduciary capacity? A) Changing the level of employer contributions B) Determining the age at which benefits are to be provided C) Selecting and monitoring third-party service providers D) Amending the plan

C Explanation The issue here is the distinction between fiduciary functions and something called settlor functions. ERISA defines fiduciary not in terms of formal title but rather in functional terms of control and authority over the plan. ERISA provides that a person is a fiduciary with respect to an employee benefit plan to the extent that such a person does any of the following: exercises any discretionary authority or control over the management of a plan or over the management or disposition of plan assets; renders investment advice for a fee or other compensation, direct or indirect, with respect to any monies or other property of such plan; or has any discretionary authority or discretionary responsibility in the administration of such plan including appointing other plan fiduciaries or selecting and monitoring third-party service providers. The other choices given in the question are known as settlor functions. The most common settlor functions are design decisions involving: establishment of the plan, defining who are the covered employees and benefits to be provided, and amending or terminating the plan. Because the likelihood of an IAR ever performing settlor functions is quite remote (usually they are done by employees of the sponsoring employer), I cannot fathom why NASAA would ask something like this on the exam, but, just in case.

The main disadvantage of a contributory defined contribution pension plan is that A) the employer contributed toward the retirement planning of the employee. B) at retirement, the client might want to use the retirement fund to generate income in retirement, possibly by purchasing an annuity. C) the actual sum an employee will receive at retirement is unknown. D) the employees can choose the amount they wish to invest.

C Explanation The liability of the employer in the defined contribution pension plan is an agreed contribution to the plan. The actual performance of the plan's investments will determine the final amount to be paid to the individual at retirement. In a contributory plan, the employee is also eligible to make contributions.

Which of the following is the beneficial owner of securities in an UTMA account? A) The guardian B) The donor C) The minor D) The custodian

C Explanation The minor is always the beneficial owner under an UTMA account. The custodian merely exercises her best judgment in handling investment decisions on the minor's behalf.

You are discussing features of qualified pension plans with a client. You state that in one type of plan "the eventual amount of pension benefits will depend upon the fund's investment performance." You must be referring to which of the following? A) Deferred compensation plan B) Defined benefit C) Defined contribution plan D) IRA

C Explanation The obligation of the employer in a defined contribution plan is to add a specified contribution to the plan. Based on the performance of the investment, the employee will receive a lump sum upon retirement. In a defined benefit plan, the ending value is pre-determined, usually based on length of service and final salary. A deferred compensation plan is not qualified and an IRA is not a pension plan.

An investment is made of $10,000. At the end of the year, $500 in nonqualifying dividends has been received and the value of the investment is $10,500. If the investor is in the 30% tax bracket, the after-tax yield is A) 5.0% B) 8.5% C) 3.5% D) 6.5%

C Explanation The only return (as far as yield is concerned) is the $500 of dividends. Remember, nonqualifying dividends do not "qualify" for the 15% rate. Subtracting 30% for taxes leaves $350 which, when divided by the $10,000 initial cost, is an after-tax yield of 3.5%. If the question had asked about total return, then the $500 unrealized profit would have been included, although there would have been no tax on it.

Although there is no specific rule requiring it, most qualified plans have an investment policy statement. For those plans that do have an IPS, it would include all of the following information EXCEPT A) how the plan measures investment performance B) the schedule for future needs of the plan C) the information in the summary plan document specified by the Department of Labor D) investment parameters to be followed by the portfolio managers

C Explanation Under the rules of the Department of Labor (DOL), one of the most important documents that participants are entitled to receive automatically when becoming a participant of an ERISA-covered retirement plan, is a summary of the plan, called the Summary Plan Description or SPD. The plan administrator is legally obligated to provide to participants, free of charge, the SPD. The Summary Plan Description is an important document that tells participants what the plan provides and how it operates. It provides information on when an employee can begin to participate in the plan, how service and benefits are calculated, when benefits become vested, when and in what form benefits are paid, and how to file a claim for benefits. However, it has nothing to do with the investment policies that will be followed by the plan's advisers. That information, such as what is shown in the other choices, is found in the IPS.

Which of the following statements regarding Roth IRAs is TRUE? A) Like traditional IRAs, Roth IRA contributions may not be made after the participant reaches age 72. B) Roth IRA withdrawals are tax free in their entirety regardless of the participant's age at withdrawal. C) Roth IRAs are not subject to the minimum distribution rules until the death of the owner/participant of the plan. D) Like traditional IRAs, Roth contribution eligibility is restricted by active participation in an employer's retirement plan.

C Explanation Unlike traditional IRAs, Roth IRAs are not subject to the minimum distribution rules regarding a participant's age (72). Rather, distributions need not be made until the death of the owner/participant. For a Roth IRA withdrawal to be entirely tax free, it must be made following a 5-year holding period after the first contribution and after the participant reaches age 59½. Effective with the SECURE Act, there are no age limitations for contributions for any retirement plan.

An investor inherits 1,000 shares of the ABC Global Growth Fund when the NAV is $9.50, the bid price is $9.00, and the ask price is $9.15. Two years later, the investor sells all shares when the NAV is $14.25, the bid is $14.50, and the ask is $14.60. What are the tax consequences of this sale? A) Long-term capital gain of $4,750 B) Long-term capital gain of $5,350 C) Long-term capital gain of $5,500 D) Long-term capital gain of $5,450

C Explanation Upon death, the beneficiary inherits closed-end funds at their bid price (what the estate could have sold them for), or $9.00 per share. The sale two years later takes place at the bid ($14.50) for a profit of $5.50 per share (times 1,000 shares). Remember, in the case of a closed-end fund, the NAV does not figure into any computations; prices are based on supply and demand and have a bid and ask price, the same as any stock. How did you know this was a closed-end company? Only in the case of a closed-end company can the ask price be lower than the NAV (ask = $9.15, NAV = $9.50).

John and Martha, both in their early 40s, were divorced on November 22, 2018. Because Martha is unemployable, the terms of the divorce require John to pay Martha $300 per month in alimony and $1,000 per month in child support for their 4 children. Given that information, which of the following statements is CORRECT? A) Martha has reportable taxable income of $12,000 for the year. B) John is able to deduct $12,000 from his taxable income. C) Martha could contribute a maximum of $6,000 this year to an IRA. D) John is able to deduct $3,600 from his taxable income.

D Explanation Alimony is considered eligible income for an IRA to Martha and tax deductible to John. (IF BEFORE 2019) Child support is neither income to Martha nor deductible to John. Because Martha receives $3,600 in alimony, that would be her maximum allowable IRA contribution. Please note: Effective January 1, 2019, there were changes to the tax treatment of alimony for all divorce agreements entered on and after that date (no changes to those already in existence). Questions on the exam (and our q-bank) reflect those changes. Therefore, for any divorce before January 1, 2019, the alimony is income to the recipient and a deduction for the payor. That means the recipient can use alimony as earned income for an IRA contribution. For any divorce after December 31, 2018, it is not income to the recipient and it is not a deduction for the payor. That means the recipient cannot consider alimony as earned income for making an IRA contribution. Be sure to check the dates on any test question.

Which of the following statements about the gift tax annual exclusion are TRUE? The annual exclusion is the amount that an individual may give to other individuals each year without incurring a gift tax. The annual exclusion is currently (2019) set at $15,000. A separate annual exclusion is available for each donee. A) II and III B) I and III C) I and II D) I, II, and III

D Explanation All these statements are true. The annual exclusion that an individual donor may give to another individual (donee) each year without incurring a gift tax is currently (2019) $15,000. A separate annual exclusion is available for each donee. So, if an individual gives $15,000 to 4 donees in 1 year, the annual exclusion will shelter all $60,000.

An investor wishes to use funds in his IRA to purchase a condominium for personal use. Under current regulations, A) real estate, like life insurance, cannot be purchased in an IRA. B) this would not be a prohibited transaction unless the investor personally used the property more than 14 days per year. C) real estate, such as a personal condominium, would be a permitted investment. D) this would be a prohibited transaction.

D Explanation An IRA may invest in real estate if it is for business purposes only. If done improperly, serious problems with the IRS can result. If it is done as a truly hands-off investment, it is unlikely that there will be an issue. However, the moment the participant derives any personal benefit from the property, it becomes a prohibited transaction.

The total return of a mutual fund is equal to A) the reinvestment of all unrealized dividend and capital gain income B) all realized and unrealized capital appreciation C) annualized fund dividends divided by the current POP D) the return attained by reinvestment of all dividend and capital gains distributions plus unrealized gains or minus unrealized losses

D Explanation As with all securities, total return includes unrealized gains or losses. In the case of mutual funds, the total return also assumes reinvestment of all dividend and capital gains distributions.

Which of the following is generally believed to present a more accurate picture of a portfolio manager's performance? A) Dollar-weighted return B) Real rate of return C) Net present value D) Time-weighted return

D Explanation Because contributions and withdrawals by the investor are not under the control of the fund manager, time-weighted return is a more reliable measure of a portfolio manager's performance. Dollar-weighted is a more reliable measure of how the investor fared. Real rate of return and net present value are not relevant to this question.

IRAs and Keogh plans are similar in each of the following ways EXCEPT A) taxes on earnings are deferred B) rollovers are allowed once every 12 months and must be completed within 60 days C) distributions without penalty may begin as early as age 59½ D) the maximum allowable cash contribution is the same

D Explanation Both IRAs and Keogh plans have maximum annual allowable contribution limits but they are significantly higher in a Keogh Plan.

Which of the following individuals is clearly eligible to make a catch-up contribution? A) Emily, who is fully vested B) Roger, who has completed 1 year of service C) Sam, who has completed 15 years of service D) Hannah, who is 55 years old

D Explanation Catch-up contributions are allowed to participants who are age 50 and over.

An investor purchased stock for $50 per share at the beginning of the year. In December, the investor liquidated his stock for $55 per share, while also receiving dividends of $2 per share during the year. Assuming an inflation rate of 3%, what is the investor's real rate of return? A) 10% B) 14% C) 4% D) 11%

D Explanation Given the fact the client liquidated his shares at a price of $55, we can conclude that he attained a 10% ($5 profit ÷ $50 initial investment) return based on capital appreciation of the stock. He also received dividends of $2 per share giving him an additional return of 4% ($2 ÷ $50). By adding these 2 percentages together, we can conclude that his total return is 14%, less an inflation rate of 3%, which would give a real rate of return of 11%.

Based on the following information, which stock is most likely to appeal to a growth investor? A) Dividend payout ratio of 65% B) P/E ratio of 8:1 C) Book value of $22 per share, current market value of $17 per share D) Dividend yield of 0.3%

D Explanation Growth investors usually seek stocks with high-growth expectations, reflected by a higher-than-normal P/E ratio, typically 20:1 or higher, and a low dividend yield. usually caused by a low dividend payout ratio. It would be unlikely to find a growth stock selling for close to its book value and certainly not below it.

Which of the following regarding a Roth IRA are TRUE? The contributions are nondeductible. One may not contribute to a Roth IRA if concurrently contributing to a traditional IRA. The contributions are deductible. Withdrawals after age 59½ may be tax free. A) III and IV B) II and III C) I and II D) I and IV

D Explanation In a Roth IRA, contributions are not deductible from current income. Withdrawals after age 59½ are tax free, provided the account has been open for at least 5 years. One may maintain both a Roth and a traditional IRA concurrently. However, the maximum total contribution between both plans is $6,000 (or $7,000 for those age 50 or older).

You have a client who is switching jobs. The HR department of the new company delivered a beautiful brochure describing all the benefits offered to employees. One of these is a noncontributory money purchase pension plan. When asked by your client for an explanation, you would reply that this plan has mandatory contributions of A) both the employer and the employee. B) the employer and optional employee contributions. C) the employee but none for the employer. D) the employer but no employee contributions.

D Explanation In a contributory plan, both the employer and employee make contributions to the account. In a noncontributory plan, only the employer makes the contributions.

Ebony sets up a revocable trust, naming her daughter, Sylvia, as the sole beneficiary. Ebony has appointed the Pacific Atlantic Trust Institution (PATI) as the trustee. Any distributed income will be taxable to A) the trustee B) the trust C) the beneficiary D) the grantor

D Explanation In almost all cases, income received into a revocable (grantor) trust, whether distributed or not, is taxable to the grantor. Things are different when the trust is irrevocable, but much more complicated and not likely to be tested.

Which of the following statements are TRUE about both an individual Roth IRA and a Roth 401(k) plan? Contributions are made with after-tax dollars. One must have AGI below a certain level in order to maintain either Roth. If all the conditions are met, withdrawals are tax free. There are no RMDs at age 72. A) II and IV B) III and IV C) I and II D) I and III

D Explanation In any Roth plan, contributions are made with after-tax dollars, and assuming all conditions are met, withdrawals are tax-free. However, unlike the individual Roth IRA, there are no earnings restrictions on participants in a Roth 401(k) plan and RMDs must begin at age 72.

Tammy Jones is retiring from her company next month on her 62nd birthday. Her 401(k) has $300,000 and offers her 4 different mutual funds. After calculating what she will receive from Social Security, she concludes that she will need an additional $500 a month to retain her current lifestyle. Which of the following would be the most appropriate recommendation? A) Leave the money in her current 401(k) account B) Take a lump-sum distribution of the entire $300,000 C) Roll the money into a mutual fund withdrawal plan D) Roll the money into a traditional IRA

D Explanation It would benefit Ms. Jones most to roll the money into a traditional IRA. By doing this she would defer paying taxes on the $300,000—something she could not avoid if she took the lump-sum distribution or rolled the money into a mutual fund withdrawal plan. Although the decision to roll over into a self-directed IRA or leave the funds in the 401(k), if permitted, is one worthy of consideration, with only 4 mutual funds being offered in Ms. Jones's 401(k) account, most would agree that the increased options available in the IRA make that the better choice.

One of the ways that individuals can accumulate funds for retirement is through individual retirement arrangements (IRAs). There are a wide range of investments eligible for inclusion in an IRA and would include all of the following except A) fixed annuity contracts. B) exchange-traded funds. C) specified collectibles. D) life insurance contracts.

D Explanation Life insurance policies are prohibited investments in an IRA and, in general, collectibles are prohibited as well. There are some important exceptions to the collectible prohibition. The IRS states that an IRA can invest in one, one-half, one-quarter, or one-tenth ounce U.S. gold coins, or one ounce silver coins minted by the Treasury Department. It can also invest in certain platinum coins and certain gold, silver, palladium, and platinum bullion.

Which of the following sources of income is eligible for funding an IRA? A) Income received as a shareholder of an S corporation B) Income received as a limited partner C) Child support D) Income received as a sole proprietor

D Explanation Only earned income can be used to fund an IRA. When a business is run as a sole proprietorship, the income reported on the taxpayer's Schedule C is considered earned. As a shareholder of an S corporation or a limited partner in a partnership, the income received is considered passive rather than earned and may not be used to fund an IRA. Child support is not earned income and alimony received from a divorce settlement dated January 1, 2019 or later is also not considered earned.

It would be least likely for dividends paid on which of the following investments to meet the requirements to be considered qualified? A) Common stock B) Preferred stock C) Equity mutual funds D) REITs

D Explanation Qualified dividends are those eligible for the reduced income tax rates. Those rates can be as low as 0% and as high as 23.8%, with most falling within the 15% or 20% bracket. We don't expect the exam to test on the requirements for a dividend to be considered qualified or how you reach that 23.8% rate. You may be asked about REITs, and their dividends do not meet the definition. In similar fashion, dividends on bond funds and money market funds are not qualified because the majority of those dividends represent interest earned by the fund.

An investor purchases 1,000 shares of ABC at $42 per share. One year later, the stock is trading at $50 per share and the investor receives 50 shares of ABC as a stock dividend. How will this dividend be currently taxed? A) As a $2,100 capital gain B) As a $2,500 capital gain C) As $2,500 ordinary income D) The shares are not subject to taxation

D Explanation Shares received per a stock dividend are not currently taxable. Instead, shareholders who receive stock dividends must adjust their cost basis in the shares downward. The total number of new shares, multiplied by their new adjusted basis, must equal the shareholder's total interest before the stock dividend was received.

In order to compute an investor's after-tax return on a corporate bond, all of the following are necessary EXCEPT A) interest payments B) marginal tax bracket C) appreciation D) inflation rate

D Explanation The after-tax return is computed by taking the total return (appreciation plus income) and taking the investor's tax rate into consideration. The inflation rate is necessary for the inflation-adjusted or real rate of return.

In a trust, the person who establishes the trust and decides on its terms is A) the trustee B) the fiduciary C) the beneficiary D) the grantor

D Explanation The grantor, sometimes called the settlor, is the person who establishes the trust and specifies its terms. The person who administers the trust is the trustee, and the person who receives distributions from the trust is the beneficiary. Interestingly, trust law would permit the grantor to also be the beneficiary and/or the trustee.

Which of the following statements regarding a traditional IRA for someone filing a 2021 tax return is TRUE? A) Distributions before age 59½ are subject to a 10% penalty in lieu of income taxes. B) Distributions without penalty may begin after age 59½ and must begin by April 1 of the year preceding the year an individual turns 72. C) With sufficient earned income, a taxpayer who contributes $6,000 to a Roth IRA can also contribute $6,000 to a traditional IRA. D) The income and capital gains earned in the account are tax deferred until the funds are withdrawn.

D Explanation The income and capital gains earned in the account are tax deferred until the funds are withdrawn. A traditional IRA allows a maximum annual contribution of $6,000 per individual ($7,000 for those age 50 and older). One may contribute to both a Roth IRA and a traditional IRA, but the total contribution cannot exceed the limit for a single IRA ($6,000 or $7,000 for age 50 and over). Distributions without penalty may begin after age 59½ and must begin by April 1 of the year following the year an individual turns 72. Distributions before age 59½ are subject to a 10% penalty in addition to ordinary income tax.

Parker and Mary have recently divorced. For Mary to receive Social Security benefits based on Parker's earnings, which of the following conditions must exist? A) The marriage must have lasted at least 10 years. B) Parker must already be at full retirement age. C) Parker must not be remarried. D) Mary must have worked at least 40 quarters to be eligible for benefits.

D Explanation The marriage of these two must have lasted at least 10 years. In addition, Mary cannot be remarried (Parker can be). It is Parker who must have at least 40 quarters to earn Social Security benefits. As long as Parker is drawing benefits, and one can start before full retirement age, benefits will be available as long as all of the conditions are met.

Since its inception in 1986, virtually all the states have replaced the Uniform Gifts to Minors Act with the Uniform Transfers to Minors Act. It is generally agreed that one of the primary benefits offered by UTMA over UGMA is A) greater flexibility in naming custodians B) mandatory surrender of control at majority C) greater flexibility in naming beneficiaries D) greater flexibility in the type of property that may be transferred

D Explanation The property that may be transferred into an UGMA account is generally limited to cash and securities, while in an UTMA account, almost any kind of property—real or personal, tangible or intangible—can be transferred to the custodian. UTMA (T=Treasure = MORE PROPERTY)

There are several financial models that refer to the "risk-free" rate of return. Which of the following instruments is used to measure that rate? A) Federal funds B) 30-year Treasury bond C) 1-year CD D) 91-day Treasury bill

D Explanation The standard benchmark used to measure the "risk-free" rate of return is the 91-day Treasury bill.

The true rate of return on a bond is the nominal rate minus A) the discount rate B) the prime rate C) the margin rate D) the inflation rate

D Explanation The true rate of return (another way of saying real rate of return) on a bond considers the inflation rate because inflation reduces the purchasing power of the interest payments.

Which of the following is not a market cap-weighted index? A) FTSE 100 and FTSE All-Share B) Dow Jones Industrial Average C) S&P 500 D) Morgan Stanley Capital International

Explanation The Dow Jones Industrial Average is a price-weighted index. All other options are market cap-weighted indexes. The PRICE of Dough (dow)

Which of the following has the power to close a stock exchange for up to 90 days? A) The Administrator in the state where that stock exchange is located B) The president of the United States C) The president of that stock exchange D) The SEC

Explanation The Securities Exchange Act of 1934 granted the SEC the power to close any registered stock exchange for up to 90 days. All that is required is notice to the president of the United States.

Which of the following statements regarding participant loans in a 401(k) plan are CORRECT? The maximum allowable loan amount is the lesser of $50,000 or 50% of the participant's vested account balance. Unless the loan is taken out for the purpose of a mortgage on the participant's principal residence, repayment must be completed within 60 months of obtaining the loan. Payback of the loan will be through payroll deduction. Default on the loan will result in the IRS treating the loan as a distribution. A) I, II, III, and IV B) II and III C) I and II D) III and IV

A Explanation All these statements regarding borrowing from a 401(k) plan are true.

All of these would be characteristics of a traditional 401(k) plan EXCEPT A) the employer can contribute more than 25% of total payroll B) employees may have a portion of their contribution matched by the employer C) employees can choose from a variety of investment options D) in-service employees may be eligible for hardship withdrawals

A Explanation 401(k) plans provide for hardship withdrawals, a choice of investment options, and employer matching. Although there are exceptions to this, in general (and on the exam), you will have to know that the employer share of the contributions to a traditional 401(k) plan (or any other DC plan) may not exceed 25% of total payroll.

If an employed client has $12,000 of capital gains and $15,000 of capital losses in the most recent taxable year, how much unused loss, if any, is carried forward by the client to the following tax year? A) $0.00 B) $12,000.00 C) $15,000.00 D) $3,000.00

A Explanation After netting the $12,000 capital gains and $15,000 capital losses, the client has a net capital loss of $3,000. Because all this capital loss may be used to offset ordinary income in any one taxable year, there is no amount of loss to carry forward. 12-15=3 loss 3 loss taken for year = none for carry

Which of the following circumstances may cause a person to be identified as a fiduciary? Investment adviser representative who becomes a trustee Investment adviser representative who becomes a member of the board of directors of a foundation Investment adviser representative who holds himself out as a fiduciary for ERISA plans and pensions Investment adviser representative who manages a discretionary account A) I, II, III, and IV B) I, II, and IV C) I and IV D) II and III

A Explanation All of these statements are correct. Trustees, board members of a foundation, IARs who exercise and those who hold themselves out as a fiduciary for an ERISA plan will generally find themselves being defined as a fiduciary.

The type of trust created by a will that becomes operative at death is A) a testamentary trust B) a living trust C) a revocable trust D) a Q-tip trust

A Explanation As in "last will and testament."

Which of the following securities can legally be purchased on margin? A) Warrants exercisable into shares of a stock traded on the Nasdaq Stock Market B) Units of a variable annuity issued by an insurance company authorized to do business in the state C) Put options where the underlying common stock is listed on the New York Stock Exchange D) Preemptive rights to acquire shares of a stock traded on the New York Stock Exchange

A Explanation Warrants, but not rights, may be purchased on margin if the underlying stock is traded on the NYSE or the Nasdaq. No insurance product is marginable and the same is true for options. LEAPS are option contracts that are marginable, but the question would have to specify that the contract was a LEAPS.

You have a client who wishes to manage his own portfolio of individual stocks. The simplest style for him to follow would be A) buy and hold B) tactical C) core D) indexing

A Explanation When it comes to individual stocks, nothing is simpler than buy and hold. If the client wished to have the simplest overall portfolio and didn't want to manage things, then indexing would be the answer.

For purposes of the maximum allowable annual contribution, an individual would have to aggregate contributions made to A) a 401(k) and a 403(b). B) a 401(k) and a 457. C) a 401(k) and a Roth IRA. D) a 403(b) and a 457.

A maintaining a 401(k) and a 403(b) is similar to maintaining a Traditional and Roth IRA. The maximum is not doubled, it is aggregated.

Employee contributions to a 401(k) plan are subject to Social Security taxes federal unemployment taxes federal income tax withholding state income tax withholding A) II and IV B) I and II C) III and IV D) I and III

B Employee contributions are excluded from taxable income at the time of contributions, which exempts them from income tax, but not from payroll taxes.

Which of the following would be a common use of a stop order? To protect the profit on a long position To prevent loss in a short position To buy at a specific price guaranteed by a specialist To lock in a price with the specialist A) II and III B) I and II C) II and IV D) I and III

B Explanation A buy stop may protect an investor against losses in a short position a sell stop may protect an investor's profits on stock she holds long.

Which of the following would not constitute a conflict of interest between the plan and a fiduciary? A) A fiduciary sells a real estate investment to the plan at the current market rate. B) The fiduciary receives fees for acting as a trustee to the plan. C) A fiduciary participates in a transaction on the plan's behalf that involves a party with interests adverse to those of the plan in order to ensure favorable terms for the plan. D) A fiduciary offers reduced commissions to the plan for transactions that are executed through his employing financial institution.

B Explanation A fiduciary can receive compensation from the sponsor of the plan for acting as a trustee, if fees are reasonable and consistent with duties performed. A fiduciary may not sell a real estate investment to the plan at the going market rate. Such self-dealing presents a conflict of interest regardless of the terms of the transaction. A fiduciary may not participate in a transaction on the plan's behalf that involves a party with interests adverse to those of the plan in order to ensure favorable terms for the plan. The situation is self-dealing and presents a conflict of interest prohibited under ERISA. Offers of reduced commissions to the plan for transactions that are executed through his employing financial institution are prohibited and a conflict of interest.

A stop order can be used to do all of the following EXCEPT A) protect a long position B) give the broker-dealer discretion regarding time and price C) protect a short position D) become a market order once the security trades at or through the stop price

B Explanation A stop order becomes a market order when the stop price is reached or penetrated. Stops can be used to protect both long and short positions. The stop order does not give the agent or the firm any discretion.

A client enters an order as follows: Sell stop 100 shares of LTC at 45 limit 45.50. Following the entry of that order, trades occur in the following sequence: 47; 46; 45.12; 44.97; 45.28; 45.97; 46.05. More than likely, the client received A) 44.97 B) 45.97 C) 46.05 D) 45.28

B Explanation This is really two orders. The first is to stop at 45. That is, once the stock trades at 45 or lower, enter the order. The second order is a sell, but with a limit of 45.50. So the first time the stock hits 45 (or less) is the trade at 44.97. That triggers the sell limit. The next trade is at 45.28 and that is not acceptable to the limit order at 45.50. Because the limit order is saying, "get me 45.50 or higher," the 45.97 is an acceptable price.

During your annual review with your clients, Matt and Sally Eberhart, they indicate that they think it is time to start putting away some money for college for their 3-year-old son. They ask you to describe the advantage of using an UTMA account over a Coverdell ESA. You would likely point out all of the following as advantages EXCEPT A) there is no limit to the amount that can be contributed to an UTMA B) contributions to the UTMA are made with after-tax dollars C) there are no earnings limits for making UTMA contributions D) withdrawals for other than qualified education expenses are not subject to any penalties

B Explanation We're looking for a feature possessed by the UTMA that is not found in an ESA, but in both cases, contributions are made with after-tax dollars. Therefore, you would not describe that as an advantage. Unlike the ESA where couples earning in excess of $220,000 per year are not eligible to contribute, no such ceiling is imposed on those donating or transferring property to an UTMA. Unlike the ESA, where there is a 10% tax penalty on the earnings withdrawn for nonqualified educational expenses, no such penalty applies to an UTMA. Unlike the ESA, which has a $2,000 per year per child limit, there is no limit to the amount that one can give to an UTMA. However, unlike the ESA, where all earnings are tax free if used for qualified educational expenses, earnings in an UTMA are taxable and, if over a certain amount, might be taxed at the parent's top marginal rate.

Under UTMA, which of the following are allowable distributions for the benefit of the minor? A) A percentage of housing expenses, such as the utilities for his bedroom B) The cost to attend a summer camp C) A percentage of food expense D) Clothing expense for a child who has gone through a growth spurt

B Explanation You cannot use UTMA (or UGMA) money for the basics: food, clothing, and shelter; those are the responsibility of the parent. An optional expense, such as summer camp, vacation, and sports league registration, would be permitted.

All of the following would be reasons for an employer to choose a nonqualified plan over a qualified plan except A) the nonqualified plan can discriminate in favor of highly-compensated employees. B) the nonqualified plan provides an immediate income tax deduction for the employer. C) the nonqualified plan provides greater flexibility. D) the nonqualified plan is not subject to ERISA reporting and disclosure requirements.

B Nonqualified plans do not provide a tax deduction to the employer until the employee receives the economic benefit as income at some point in the future. They are, however, more flexible because they do not have to comply with ERISA reporting and non-discrimination requirements.

A stock is currently worth $75. If the stock was purchased one year ago for $60, and the stock paid a $1.50 dividend over the course of the year, what is the holding period return? A) 22.0% B) 25.0% C) 27.5% D) 24.0%

C Explanation (75 − 60 + 1.50) ÷ 60 = 0.2750, or 27.5%.

Which of the following statements about 401(k) plans are CORRECT? 401(k) plans are a type of defined benefit retirement plan. An employee's elective deferrals are made with pre-tax dollars. Earnings on the contributions to a 401(k) accumulate on a tax-deferred basis. A) I, II, and III B) I and II C) II and III D) I and III

C Explanation A 401(k) plan is a type of defined contribution plan rather than a defined benefit plan. A participating employee is not guaranteed a specific retirement benefit from the plan; instead, the retirement benefits depend on how much is contributed to the plan and how much the contributions earn. A 401(k) plan allows employees to make elective deferrals from their pay which results in those funds being contributed on a pre-tax basis. Plan earnings accumulate on a tax-deferred basis. This means that plan participants do not pay income tax on the earnings until they are withdrawn.

One of your clients currently holds a short position in DEF common stock. Which of the following types of orders is designed to offer the client protection against loss? A) Buy limit B) Sell limit C) Buy stop D) Sell stop

C Explanation The risk to a short seller is to the upside (there is, at least theoretically, no limit as to how high the stock's price can go). To protect against an increase to the stock's price beyond the point the investor is willing to lose, it is wise to enter a buy stop order at that price. If the stock should reach that price, the order is triggered, a market order is entered, and the short position is closed out. This is why stop orders are usually referred to as stop-loss orders; they keep you from losing any more money.

KapCo Balance Fund has a NAV of $9.50 and POP of $10. Over the past 12 months, it distributed dividends totaling $.75 and capital gains totaling $1.00. What is KapCo's current yield? A) 10% B) 17.5% C) 7.5% D) 7.9%

C mutual fund CY: Dividends Divided by POP Explanation This question gives you excess information. The first point is that capital gains are not included in calculation of a mutual fund's current yield. You must also remember that the NAV is not involved. The calculation is: $0.75 (annual dividend) = 7.5% ------------------------------- $10.00 (POP)

Mr. Adam Samuels suffers a massive heart attack and dies at the age of 62. As part of his estate, there is an IRA with a current value of $170,000. A review of the IRA documents reveals that Mrs. Eve Samuels, the wife, is the primary beneficiary and their 2 children have been named as contingent beneficiaries. Eve is 50 years old and does not need the income from the IRA and would like to preserve the IRA for her children to inherit. Which of the following steps would you recommend Mrs. Samuels take? A) Cash in the IRA because as a spouse of a deceased, she will avoid the 10% tax penalty. B) Disclaim the IRA and let it pass to the contingent beneficiaries. C) Execute a rollover into an IRA in her name. D) Execute a rollover into an inherited IRA.

C Explanation This is a highly complicated question and there is room for disagreement. However, if a question similar to this were to appear on your exam, the answer selected is the one that NASAA would mark as the correct one on its test. The key to this question is the word "preserve." By executing a rollover into an IRA in her name, tax deferral of the assets continues and RMDs are not required until after Mrs. Samuels turns 72. Thus, the assets are preserved for at least 20+ years. If she took the distribution, she would not have to pay the penalty tax, but there would be ordinary income tax due and this would not meet her objective of preservation of the IRA. If she disclaimed, the assets would then go to the children, but they would have to begin taking RMDs over a 10-year period. Not a bad choice, but the assets are being distributed and taxed, not preserved. The benefit of rolling over into an inherited IRA (sometimes called a beneficiary IRA) instead of one in her own name is that she can begin taking distributions right now without the 10% penalty, even though she is only 50. However, the question stated that she did not need the income, and RMDs must begin at the time they would have been required for Mr. Samuels, 12 years earlier than if she chooses to rollover into her own IRA.

Which of the following statements regarding grantor trusts is NOT correct? A) If the grantor has the power to revoke the trust, he is treated as the owner of the trust. B) If the grantor can control the beneficial enjoyment of the trust, he is treated as the owner of the trust. C) If the grantor can receive income from the trust, he is treated as the owner of the trust. D) The grantor may be taxed on trust income only if the grantor actually received the income.

D Explanation As long as the grantor has the power directly or indirectly to control the trust, he is treated as the owner. The grantor may be taxed on trust income if the grantor either actually or constructively receives the income.

A single individual earning $250,000 a year may open a Coverdell ESA not open a Coverdell ESA open a 529 college savings plan not open a 529 college savings plan A) II and IV B) I and III C) I and IV D) II and III

D Coverdell NOT COVERED Explanation There are income limits that apply to Coverdell ESAs. Single individuals earning more than $110,000 per year are not permitted to open a Coverdell account, and married couples lose the ability to contribute when earnings exceed $220,000. However, there are no income limits restricting who is eligible to open and contribute to a Section 529 college savings plan.

Which of the following statements about plan fiduciaries under ERISA are TRUE? Plan fiduciaries sometimes have conflicting obligations to plan participants and other parties in interest. Plan fiduciaries must ordinarily diversify plan investments. Plan fiduciaries are personally liable for fines if they violate their fiduciary duties. A) I, II, and III B) I and III C) I and II D) II and III

D Explanation Under ERISA, plan fiduciaries must act solely in the interests of plan participants and beneficiaries, and they may not place the interests of other interested parties above those of the plan participants and beneficiaries. They must diversify plan investments to minimize the risk of large losses, unless it would not be wise to do so. If they violate any of their fiduciary duties, they may be personally liable for large fines.

Under the Uniform Gifts to Minors Act, Ralph wants to give some stock to his brother's son, Jose. His nephew's father, Bob, is the legal guardian. If Ralph wants to name himself as custodian, which of the following needs to be done? A) Ralph must file the proper legal documents. B) Ralph must have the permission of the guardian. C) Ralph must receive legal permission to act as custodian. D) Ralph must open the account and name himself as the custodian.

D Explanation Under UTMA or UGMA, no special documentation is required. The account is opened in the name of the minor with the minor's Social Security number and the name of the adult listed as custodian.

Which of the following is the primary advantage to the employer who offers a nonqualified plan when compared to one that offers a qualified plan? A) The qualified plan costs less to administer than the nonqualified plan. B) The qualified plan is permitted to discriminate in favor of key employees. C) The nonqualified plan allows for an immediate employer deduction for contributions. D) The nonqualified plan is permitted to discriminate in favor of highly compensated employees.

D Explanation Unlike a qualified plan, a nonqualified plan is permitted to discriminate in favor of highly compensated employees.

One of your clients has a margin account. There is a drop in the value of the stock owned in the account, and additional funds are required based on the terms of the firm's margin agreement. This would be known as A) a sellout B) a Regulation T call C) a margin call D) a house call

D Explanation When additional funds are required, it is known as a house or maintenance call. If based on the firm's stricter requirement, it is a house call; if based on the requirement of the SRO, it is a maintenance call. The initial or Regulation T call occurs at the time of the purchase. If any call for funds is not met, then there will be a sellout.

A client of a broker-dealer who sold 200 shares of XYZ stock short, A) must repay the money borrowed to effect the short sale. B) might consider purchasing put options on XYZ as protection. C) has a bullish outlook on XYZ stock. D) is obligated to return the 200 shares borrowed.

D Return what is borrowed

Denise is an expert in retirement planning working with a client to estimate the client's inflation-adjusted retirement funding. If in today's terms, the annual retirement income needed by the client is $15,000. What would be the approximate amount required after three years, given an inflation rate of 4%? A) $14,768 B) $11,456 C) $19,567 D) $16,872

D Without compounding, that is $600 per year ($15,000 x 4% = $600). Three years at $600 per year is a total of $1,800 additional needed. That brings the client's needs to $16,800. That doesn't include the compounding so the answer will be slightly higher (and that always works on the exam).

The current market interest rate for a bond rated AA with 20 years to maturity is 5%. In an efficient market, a similar bond with a coupon of 4% could be expected to have an internal rate of return of A) 4%. B) 5%. C) 8%. D) 6%.

Explanation In an efficient market, bonds are priced so that their NPV is zero. That means the bond's yield to maturity is equal to the current market interest rates for similar bonds. When that rate is 5%, as is given in this question, all AA bonds with 20 years remaining to maturity should have a YTM of 5%.

A client purchases 1,000 shares of the ABC Global Growth Fund when the NAV is $8.75 and the POP is $9.21. Three years later, the client makes a gift to her daughter when NAV is $9.50 and POP is $10.00, and the daughter elects to receive all distributions in cash. Two years later, she sells all shares when the NAV is $14.25 and POP is $15.00. What are the tax consequences of this sale? A) Long-term capital gain of $4,750 B) Long-term capital gain of $5,040 C) Long-term capital gain of $5,000 D) Long-term capital gain of $5,500

Explanation In the case of a gift of securities, the donee acquires the donor's cost basis, $9.21 per share. Sale (redemption) takes place at the NAV ($14.25) for a profit of $5.04 per share (times 1,000 shares). Gift= original POP Sale= END NAV

If the return on Treasury bills is 3% and the equity risk premium is 4%, the expected equity returns should be A) 1% B) 12% C) 4% D) 7%

D Explanation The expected return on an equity investment is the risk-free (for example, T-bill) rate of return added to the equity risk premium (3% + 4% = 7%).

Jimmy Merchant is an agent with FLATT securities, a registered broker-dealer. When Jimmy submits an order ticket to purchase securities for a client, all of the following would appear EXCEPT A) the current market price of the security B) the broker-dealer's name C) the account number D) Jimmy's name

A Explanation Any order ticket submitted by an agent for execution at a broker-dealer will always include the account number, the agent's name, and that of the BD. All order details must be listed (e.g., the number of shares, limit or market, etc.), but the current market price is never included. Once the order is executed, the execution price is entered.

A married couple has lived in the same home for 40 years and now, with the children all gone, they've decided to sell and move to a retirement village. They purchased the home for $80,000 and have accepted a contract for $800,000. The tax consequences of this sale is A) $220,000 capital gain. B) $470,000 capital gain. C) $720,000 capital gain. D) $0 capital gain.

A Explanation As long as a homeowner has lived in the primary residence at least 2 of the previous 5 years, the first $250,000 of profit on a home sale is excluded from tax. In the event it is a married couple, as in this question, the exclusion is doubled to $500,000. The profit on the sale was $720,000 ($800,000 minus the cost of $80,000) and the exclusion of $500,000 reduces the reportable gain to $220,000.

If the Consumer Price Index (CPI) rose 5% during the past year, during which time your client held a 6% bond, what would be the approximate annualized inflation-adjusted return? A) 1% B) 5% C) 0% D) 6%

A Explanation Because inflation, as measured by the CPI, rose by 5% during the year and the client's bonds returned 6% annually, inflation would have reduced the client's purchasing power by 5%, leaving an inflation-adjusted return of 1% for that year.

Which of the following employer-sponsored plans allows coverage to discriminate in favor of key employees? A) 457 plan B) 401(k) plan C) 403(b) plan D) Defined benefit pension plan

A Explanation Because the 457 plan is technically non-qualified, it does not come under the non-discrimination rules of ERISA.

In the securities industry, the term contra party refers to A) the person on the other side of the trade B) the person on the other side of a civil suit C) a securities regulator who begins an investigation against a securities professional D) the person identified on the trade confirmation as a broker

A Explanation Contra party is defined as the broker-dealer or customer to whom a person has sold securities or from whom a person has purchased securities—they are on the other side of the trade.

A company has paid a dividend every quarter for the past 20 years. If the stock's price has fallen dramatically over the past quarter, but the dividend has remained the same, it may be concluded that A) current dividend yield has increased B) dividend yield to maturity has decreased C) current dividend yield has remained the same D) current dividend yield has decreased

A Explanation Current dividend yield is income dividend divided by price. If the price of a stock decreases and the dividend remains the same, dividend yield will increase. DDP

Investors looking to minimize the effects of taxation on their investments would probably receive the least benefit from A) a corporate bond B) an S&P 500 index fund C) an apartment building D) a growth stock

A Explanation Investors receive interest income from corporate bonds. That income is fully taxable at ordinary income rates. Real estate ownership has certain tax benefits, such as depreciation and a deduction for operating expenses. Index funds are known for their high tax efficiency and investors in growth stocks anticipate long-term capital gains which are taxed at a lower rate than ordinary income.

A QDRO is a judgment, decree, or order for a qualified retirement plan to pay child support, alimony, or marital property rights to a spouse, former spouse, child, or other dependent of a participant. The QDRO must contain certain specific information as stated in whose regulations? A) IRS B) NASAA C) DOL D) ERISA

A Explanation It is the IRS who states the QDRO must contain certain specific information, such as: the participant and each alternate payee's name and last known mailing address, and the amount or percentage of the participant's benefits to be paid to each alternate payee. This is not part of ERISA or the Department of Labor and, least of all, NASAA.

An agent of a broker-dealer has a client who lost her job but will be starting a new job in 3 weeks. The client is in need of $900 for the 3-week gap. Under what circumstances may the agent arrange a loan for the client? A) If the client has $5,000 in her brokerage account B) If the client is agent's niece C) If the loan is repaid within 30 days D) If the loan is less than $1,000

A Explanation Loans may be made to clients if the person making the loan is in the lending business. Broker-dealers are permitted to lend money against securities held in client's portfolios. This is known as a margin loan. In fact, with $5,000 in the account, current regulations would permit a loan of up to $2,500

Which of the following items are NOT included in the gross estate of a decedent? A) Proceeds from a life insurance policy owned by the deceased's spouse B) Property held in an account registered tenants in common C) The first $250,000 of a primary residence if owned singly, $500,000 if owned jointly with spouse D) Proceeds from a life insurance policy held in a revocable trust

A Explanation One popular estate-planning technique is to have life insurance owned by (and premiums paid by) someone other than the insured. In that case, proceeds are generally excluded from the gross estate of the deceased. If the trust was irrevocable, that same benefit might be achieved, but not with one that is revocable. There is an exclusion for income tax purposes on the sale of a primary residence, but that has nothing to do with the estate. Finally, when property is owned tenants in common, the percentage belonging to the deceased is part of the gross estate.

A client owns 300 shares of BACH common stock in a margin account. The stock was originally purchased at a price of $40 per share and the Reg. T call was met. If the BACH is now selling for $50 per share, disregarding interest charges, the client's equity is now A) $9,000 B) $6,000 C) $1,000 D) $3,000

A Explanation Purchasing 300 shares at $40 per share is a total of $12,000. The Reg. T call of 50% requires a deposit of $6,000 with the remaining $6,000 the loan from the broker-dealer. If the market price of the shares increases to $50, the current market value of the account is $15,000. With a debit balance (the amount borrowed from the BD) of $6,000, the equity is $9,000. If you answered $3,000, you probably forgot the investor owned 300 shares, not 100. 12 divided by 2 = 6 15-6=9

Each of the following are advantages offered by a nonqualified deferred compensation plan that are not found in a qualified plan EXCEPT A) employer contributions to the plan are not subject to current taxation to the employee. B) they are an attractive benefit to the employer because participation requirements and nondiscrimination restrictions do not apply. C) deferred compensation plans are not subject to most of the requirements of the Employee Retirement Income and Security Act of 1974 (ERISA). D) they are an attractive benefit for highly compensated employees because they're free from the contribution limits.

A Explanation Tax deferral is found in both NQDC plans and qualified plans, so there is no advantage that one has over the other. However, NQDC plans have much more flexibility without the burdensome compliance issues with ERISA.

A client owns a taxable bond with a coupon rate of 5%. His marginal tax rate is 28%. What is the after-tax yield he will receive on this investment? A) 3.60% B) 1.40% C) 6.40% D) 6.94%

A Explanation The client will earn an after-tax yield of 3.60%, or 5% × (1 − 0.28).

A customer purchases stock for $40 per share and holds it for 1 year, selling it for $50 per share exactly 12 months after the date of purchase. Four quarterly qualifying dividends of $.50 were paid during the year. If the customer's tax bracket is 30%, what is the after-tax rate of return? A) 21.75% B) 21% C) 17.5% D) 18.40%

A Explanation The customer's return on the stock includes the $10 per share short-term capital gain ($50 − $40) plus the $2 qualifying dividend (quarterly dividend of $0.50 × 4). Remember, an asset must be held for more than 12 months for the gain to be long-term. After-tax rate of return is found by computing the total after-tax earnings. Short-term gains are taxed at the same rate as ordinary income, and qualifying dividends are taxed at a maximum rate of 15% (except for very high income earners—not tested). The tax on the $10 gain is $3 ($10 × 30%), and the tax on the $2 qualifying dividend is $0.30 ($2 × 15%). The investor's total return is the $12 total minus the $3.30 in taxes, or $8.70; $8.70 divided by the original investment of $40 results in an after-tax return of 21.75%.

What is the name given to an order to purchase or sell a stock where the investor has specified a price? A) A limit order B) A market order C) An all or none order D) A discretionary order

A Explanation The distinguishing feature of a limit order is that the investor sets a specific price limit. In the case of a buy limit, it is the maximum price he is willing to pay; in the case of a sell limit, it is the lowest price he is willing to accept. Although an all or none order does specify a price, it is categorized as a type of limit order so you must choose the most all-encompassing answer.

Where would you be most likely to find an IPS? A) Defined benefit plan B) SPD C) IRA D) GRAT

A Explanation The investment policy statement (IPS), although not required under Department of Labor (DOL) rules, is generally found in corporate qualified plans, such as the defined benefit or defined contribution plan. Because the investor manages the IRA, there is no need to prepare an IPS for participants to review.

A company currently has earnings of $4 and pays a $0.50 quarterly dividend. If the market price is $40, what is the current yield? A) 5% B) 15% C) 10% D) 1.25%

A Explanation The quarterly dividend is $0.50, so the annual dividend is $2.00; $2 ÷ $40 (market price) = 5% annual yield (current yield).

William died in 2019 with the following assets and liabilities: $200,000 in securities left to his wife, $650,000 home left to his wife (the home cost $150,000), $250,000 life insurance policy with his daughter named as beneficiary, and $75,000 in debts and estate expenses. What is William's net estate? A) $175,000 B) $750,000 C) $0; it is below the $11.4 million exemption equivalent D) $625,000

A Explanation The question is asking for the net estate, not the amount of estate tax due. The market value of all assets that William has an incident of ownership in will be included in the gross estate. All assets left to the spouse and the debts/expenses are allowable reductions to arrive at the net or taxable estate. The math goes like this. The $1.1 million gross estate (add together the assets ($200,000 + $650,000 + $250,000) is reduced by the $850,000 left to his wife. That brings the net estate down to $250,000 ($1,100,000 minus $850,000). The net estate is further reduced by the $75,000 in debt and expenses. Subtracting $75,000 from $250,000 leaves a net estate of $175,000. That is well below the estate tax exemption of $11.4 million in assets for 2019.

One of your clients has just completed a divorce. Under the terms of a QDRO, half of the client's traditional IRA now belongs to the ex-spouse. Which of the following statements is correct? A) Withdrawals prior to age 59½ may be subject to the 10% penalty. B) The name of the former spouse must appear on the ex-spouse's IRA. C) The ex-spouse has 60 days to rollover the distribution. D) The ex-spouse will be required to take RMDs based on the life expectancy of the former spouse.

A Explanation The simple explanation is QDROs only apply to qualified plans and, therefore, if ex-spouse withdraws funds prior to age 59½ (unless due to one of the allowable exceptions), the 10% tax penalty applies. When there is a divorce and an IRA is split, the ex-spouse now has an IRA in his or her name with no mention of the previous owner. There is technically no distribution so there is nothing to rollover.

When discussing a stock exchange, a specialist is A) an electronic brokerage concern that executes trades online and through specialized trading order executing services B) a member of the New York Stock Exchange who executes orders for other members and who also acts as a market maker charged with the responsibility of keeping an orderly market in designated stocks C) a floor broker on the New York Stock Exchange who only executes trades for other brokers in return for commissions D) a trader who makes a market in OTC stocks and ADRs

B Explanation A specialist is a member of the NYSE who executes orders for other members and who also acts as a market maker charged with the responsibility of keeping an orderly market in designated stocks. A specialist must have sufficient capital to buy and sell from his own account in order to maintain a liquid and orderly market. The term specialist has been replaced by designated market maker (DMM), but it seems that specialist may still be in use on the exam. A trader who makes a market in OTC stocks and ADRs is a market maker in the OTC market and not a specialist on an exchange. A specialist executes trades on an exchange.

Which of the following statements regarding a qualified profit-sharing plan is TRUE? A) Contributions are required annually. B) It must be established under a trust agreement. C) It can permit regular direct cash payouts to participants before retirement. D) It must define a specific contribution amount.

B Explanation All qualified retirement plans must be established under a trust agreement

Which of the following assets will have the greatest effect on minimizing financial assistance when an individual is applying to college and using the FAFSA application? A) A Coverdell ESA B) An UTMA account C) A Roth IRA D) A prepaid tuition plan

B Explanation Although the exact percentages will likely not be tested, 20% of the money in an UTMA (or UGMA) account is counted, while only 5.64% of a Section 529 plan (either option) is counted. Retirement accounts are not considered assets on the application for student aid, which means the value of a Roth IRA won't hurt the individual's chances for financial aid eligibility.

An investor owns a common stock that has been paying a dividend at an annual rate of $2.00. If the investor buys 100 shares of the stock at $50 and sells it 3 months later for $52, the approximate annualized rate of return is A) 5% B) 20% C) 4% D) 12%

B Explanation Annualized rate of return is computed by taking the investor's total return and annualizing it. In this case, the investor had $2 of appreciation and $0.50 (1 quarter) in dividends. Total return of $2.50 divided by the $50 cost is 5%. But, that is for 3 months − 1 quarter. Multiply that by 4 to get the annual rate.

Distributions from which of the following can be rolled over into an IRA? Another IRA Corporate pension plan Corporate profit-sharing plan Keogh plan A) III and IV B) I, II, III, and IV C) II and III D) I and IV

B Explanation Assets from any qualified corporate plan or from another IRA may be rolled over into an IRA.

One of your ultra-high net worth clients has extensive real estate holdings and is concerned about his children being forced to liquidate some of them in order to pay the estate taxes after his death. One tool that could be suggested to solve this problem would be A) registering the properties as JTWROS. B) purchasing a life insurance policy using an ILIT. C) placing the properties into a living trust. D) using a TOD account.

B Explanation Estate taxes must be paid within 9 months of death. If the client doesn't want to have to liquidate his real estate holdings, then another source for the tax payment must be found. A frequently-used tool is the irrevocable life insurance trust (ILIT) where a policy is purchased on the life of the client, but owned by the trust. When properly structured, this means that the death benefit is not included in the estate and passes tax free to the beneficiaries. Those funds can then be used to pay the estate taxes and the real estate assets pass to the beneficiaries. A living trust won't work because the only way the policy's proceeds aren't considered part of the estate is when the trust is irrevocable. TOD and JTWROS avoid probate, but do not avoid estate taxes.

Which of the following business entities has an income tax filing due date (disregarding possible extensions) of March 15? Sole proprietorship Single-member LLC Multiple-member LLC electing to be treated as a corporation​ S corporation A) II, III, and IV B) III and IV C) I and IV D) I and II

B Explanation For partnership returns (including LLCs with more than 1 member) and S corporation returns, the due date is March 15. One effect of this is that LLCs, partnerships, and S corporations all have the same filing deadline. For C corporations, the due date is the 15th day of the 4th month following the close of the corporation's year; this date is April 15 for a calendar-year filer.

One of your very generous clients has used up her lifetime gift exclusion. Continuing to make gifts, she gives $50,000 to a grandchild and $18,000 to the child of a friend. What are the tax consequences of these gifts? A) The tax rate on both gifts will be the same. B) The tax rate on the $50,000 gift will be higher than that on the $18,000 gift. C) If the children use the money for tuition at a qualified educational institution, there is no tax. D) Only the gift to the child of the friend will be taxed because one can make unlimited gifts to grandchildren.

B Explanation Gift taxes and estate taxes are progressive.

A premature distribution from an IRA would be exempt from the premature distribution penalty under all of the following circumstances EXCEPT A) to correct an excessive contribution to the IRA B) as a result of hardship C) to pay for qualifying medical expenses D) upon the death of the IRA owner

B Explanation Hardship withdrawals are not permitted from IRAs. They are a feature permitted in 401(k) plans.

One of your ultra-high net worth clients would like to give some low cost basis stock as gifts to her adult grandchildren. It would be prudent for you to tell her that A) making the gift under the Uniform Transfer to Minors Act is generally the most advantageous for the child. B) unlike an inheritance, there is no stepped-up cost basis. C) it would be wise for her to use a TOD account to avoid probate. D) for purposes of the gift tax, her cost basis will be used.

B Explanation One of the benefits of inheriting low cost basis securities is the stepped-up basis and that does not apply to gifts. Although the donor will not be the one subject to capital gains tax, it would be the right thing to do to let her know that the donees (her grandchildren) will be receiving the stock at her cost basis.

If a client in the 30% marginal income tax bracket can earn an after-tax rate of return of 7% when the estimated inflation rate during the holding period of an investment is 4%, the client's real rate of return is A) 7% B) less than 7% C) 10% D) more than 7%

B Explanation Real return reduces nominal return by an inflation factor. Thus, the client's real return must be less than 7%.

Expected return is A) the worth of future income discounted to reflect what that income is worth today B) an estimate of probable returns an investment may yield C) the one discount rate that equates the future value of an investment with its net present value D) the difference between an investment's present value and its cost

B Explanation The expected return is the estimate of probable returns that an investment may yield when taking the sum of all probabilities.

Savant Investment Managers (SIM) has a client with a long position in PQR common stock. The position has an average cost per share of $30, and with PQR currently selling at $50 per share, the client is interested in a method that will allow her to protect some of the unrealized profit without an expenditure of funds. Her representative at SIM could suggest A) buying PQR 45 put options. B) entering a sell stop order at $45. C) entering a buy stop order at $45. D) entering a sell stop order at $55.

B Explanation Sell stop orders, commonly called "stop loss" orders, are designed to halt a loss or protect a gain. These are sell orders placed below the current market that become triggered if it happens that the stock should trade at or through the specified stop price. Buying the put options would also offer the protection, but the question specified no expenditure of funds.

You have a client whose income from a real estate limited partnership is $11,000. During the same year, your client had net capital losses of $2,000 and losses from an oil and gas drilling program of $6,000. The effect of this investment activity would be to increase the client's taxable income by A) $9,000 B) $3,000 C) $5,000 D) $11,000

B Explanation The $11,000 passive income is offset by the $6,000 of passive loss giving the client $5,000 of passive income. Because capital losses up to $3,000 are deductible from taxable income, we can deduct the $2,000 in net losses giving a net increase to taxable income of $3,000.

The Sharpe ratio measures a stock's A) excess return earned compared to its systematic risk. B) excess return earned compared to its total risk. C) excess return earned compared to its unsystematic risk. D) return earned compared to its total risk.

B Explanation The Sharpe ratio is defined as a fund's excess return (fund's return exceeding the risk-free rate) divided by the total risk (standard deviation).

If GHI currently has earnings of $3 and pays an annual dividend of $1.75 and GHI's market price is $35, the current yield is A) 3% B) 5% C) 8.6% D) 1.75%

B Explanation The current yield is calculated by dividing the annual dividend by the current market value ($1.75 ÷ $35 = 5%).

During the past year, the market price of Kapco common stock has increased from $47 to $50 per share. Over that period, Kapco's earnings per share (EPS) have increased from $2.00 to $2.50 per share, and their dividend payout ratio has decreased from 50% to 40%. Based on this information, the current yield on Kapco common stock is A) 4.26% B) 2% C) 2.13% D) 6.34%

B Explanation The current yield on a stock is computed by dividing the annual dividend rate by the current market price. With EPS of $2.50 and a 40% payout ratio, the annual dividend is $1.00. This dollar divided by the current market price of $50.00 results in a current return of 2%.

Which of the following could reduce the amount that an individual may contribute to a Traditional IRA? Roth IRA contributions made for the year High income level Participation in an employer-sponsored plan Marital status A) I, II and III B) I only C) I and II D) I, II, III and IV

B Explanation The maximum annual contribution applies as a total among your Roth and your traditional IRA. So, if the maximum is $6,000 and you put $3,000 into your Roth, you could only put $3,000 into your traditional IRA. You could do a total of $7,000 if you were 50 or older.

Which of the following indices or averages is based on the prices of only 65 stocks (30 industrial, 20 transportation, and 15 utility)? A) S&P Composite B) Dow Jones Composite Average C) Value Line D) Wilshire 5,000

B Explanation The most widely quoted and oldest measures of changes in stock prices are the Dow Jones averages. They are also the smallest in terms of the number of stocks included in the averages with only 65 stocks.

An investment is made of $10,000. At the end of the year, $500 in nonqualifying dividends has been received and the value of the investment is $10,500. If the investor is in the 30% tax bracket, the after-tax yield is A) 5.0% B) 3.5% C) 6.5% D) 8.5%

B Explanation The only return (as far as yield is concerned) is the $500 of dividends. Remember, nonqualifying dividends do not "qualify" for the 15% rate. Subtracting 30% for taxes leaves $350 which, when divided by the $10,000 initial cost, is an after-tax yield of 3.5%. If the question had asked about total return, then the $500 unrealized profit would have been included, although there would have been no tax on it.

You have a client who is not covered under an employer-sponsored retirement plan and has been contributing the maximum to her traditional IRA. She has just informed you that she won $1 million in the lottery, plans to continue working, and would like to continue to contribute to her IRA. Which of the following statements is correct? A) She may continue to contribute, but her contribution will not be tax deductible. B) She may continue to contribute and her contribution will be tax deductible. C) Her income for the year exceeds the allowable limit for making a contribution. D) She may continue to contribute, but only a portion of her contribution will be tax deductible.

B Explanation The only time that there is an earnings limit is when the individual (or spouse) is covered under an employer-sponsored retirement plan. That is not the case here. It is important to note that the client intends to continue in her job because lottery winnings are not considered earned income for an IRA contribution.

Your client's wife retired as a 3rd grade teacher in 2009, where she was covered under the school system's 403(b) plan. If she resumes employment with a corporate employer, and that new employer has a 401(k) plan, is she entitled to defer RMDs from the 403(b) plan past the regular age 72 date? A) RMDs may be deferred only if the current employer offered a 403(b) plan. B) RMDs may be deferred only from the plan sponsored by the current employer. C) RMDs may never be deferred for those who were participants in a 403(b) plan. D) RMDs may be deferred as long as the individual is employed on a full-time basis.

B Explanation The rule is that you can only defer RMDs in the plan of the employer where you are currently employed. For example, assume you retire from Company A and get a job with Company B, and both companies have a 401(k) plan. You can only defer RMDs from the Company B plan, because that is your current employer; you will have to take RMDs from the Company A plan.

​Oscar and Hilda, a married couple, are collecting Social Security. They speak to their financial planner for advice on taxation of those benefits. At what level do their benefits become subject to income tax?​ A) When 50% of their benefits added to all their other income, including tax-exempt interest, exceeds $25,000 B) When 50% of their benefits added to all their other income, including tax-exempt interest, exceeds $32,000 C) When 50% of their benefits added to all their other income, excluding tax-exempt interest, exceeds $25,000 D) When 50% of their benefits added to all their other income, excluding tax-exempt interest, exceeds $32,000

B Explanation These are the current numbers used by the IRS to determine if Social Security benefits are taxable. It is interesting that the computation indirectly can cause tax-exempt interest to become taxable. Once the couple's income under this computation exceeds $44,000, 85% of it is taxable. If the question dealt with a single person, the limit would be $25,000 rather than $32,000.

A portfolio manager's performance is often measured against a benchmark such as the S&P 500. A manager whose performance beats the benchmark by taking greater risk than the S&P 500 may not have had superior returns as measured on A) a total-return basis B) a risk-adjusted basis C) an expected-return basis D) an inflation-adjusted basis

B Explanation Unless the portfolio's performance is better than the extra risk taken, the manager has not beaten the performance benchmark, the S&P 500, on a risk-adjusted basis. Risk-adjusted return is calculated by computing the Sharpe ratio. Total return comprises the yield plus the growth in value of an investment over time and is not related to risk. The expected return is an estimate of the probable return an investment may yield whereas inflation-adjusted return is the nominal return reduced by the inflation rate. Neither of these returns is related to risk. Inflation-adjusted returns are often compared to a benchmark such as the Consumer Price Index (CPI). Unadjusted rates of return are called nominal rates of return.

An investor inherits 1,000 shares of the ABC Global Growth Fund when NAV is $9.50 and POP is $10.00 and elects to receive all distributions in cash. Two years later, sells all when NAV is $14.25 and POP is $15.00. What are the tax consequences of this sale? A) Long-term capital gain of $5,500 B) Long-term capital gain of $4,750 C) Long-term capital gain of $5,000 D) Long-term capital gain of $4,250

B Explanation Upon death, the beneficiary inherits mutual funds at their NAV ($9.50). Sale (redemption) takes place at the NAV ($14.25) for a profit of $4.75 per share (times 1,000 shares).

Which of the following statements regarding a QDRO is correct? A) A QDRO applies only to assets in a traditional IRA. B) A QDRO applies to assets in a qualified employer plan and a traditional IRA. C) A QDRO applies only to assets in a qualified employer plan. D) A QDRO must comply with ERISA to be effective.

C Explanation A QDRO applies only to assets in a qualified employer plan; it would not be applicable to an IRA or a SEP. Under IRS regulations, early distributions that are taken pursuant to a qualified domestic relations order, or QDRO, are exempt from the 10% penalty. A QDRO is a court-issued order that gives someone the right to an individual's qualified plan assets, typically an ex- (or soon-to-be-ex-) spouse, and the QDRO is usually issued in the course of divorce proceedings or to satisfy child support obligations.

An advisory client of yours discusses a business project she is involved with where the partnership is using accelerated depreciation to maximize losses in the early years. It would be prudent of you to inform the client that A) accelerated depreciation leads to a reduction in the partnership's cash flow. B) a maximum of $3,000 in losses can be taken against passive income in any year. C) accelerated depreciation could trigger the alternative minimum tax. D) a maximum of $3,000 in losses can be taken against ordinary income in any year.

C Explanation Accelerated depreciation is a tax preference item and could result in requiring this client to pay the AMT. These would be passive losses and they can only be taken against passive income. There is no limit to the amount of passive loss that can be deducted against passive income. Because the most common way for a company to compute cash flow is: net income plus depreciation, the reduction to net income is zeroed out by the increased depreciation added back in.

A complex trust has the following income for the year: $1,500 in taxable interest, $2,000 in dividends (reinvested in the stock), and $3,000 in tax-exempt interest. In addition, the portfolio realized $3,500 in capital gains that were reinvested in the corpus. What is the distributable net income (DNI) for the trust? A) $1,500 B) $4,500 C) $6,500 D) $10,000

C Explanation All investment income, regardless of source, will be considered DNI and will be included in the taxable income calculation to the trust unless distributed. That portion of the DNI representing tax-exempt interest maintains its tax-free status. Reinvested capital gains are not part of a trust's DNI. The computation is: $1,500 in taxable interest + $2,000 in dividends (reinvestment means nothing here) + $3,000 in tax-exempt interest. This is a total of $6,500 of DNI. When distributed, only $3,500 will be taxable.

Which of the following could accelerate a rise in a bull market? A) Buy limit B) Sell limit C) Buy stop D) Sell stop

C Explanation Buy stop orders are placed above the market, and as prices increase, the stops are hit, creating additional buying.

Which of the following is an improper activity under the Uniform Securities Act? A) An investment adviser collects a commission on the sale of insurance products that he recommended, disclosing that a commission would be earned. B) An investment adviser charges two customers two different fees for a similar service. C) A dealer charges commissions for securities it sells from its inventory and discloses the amount of the commission to the customer. D) An investment adviser charges a customer a fee for advice leading to the sale of a security, receives a commission on the sale, and discloses the amount of the commission to the customer.

C Explanation Dealers who act as principals in transactions charge markups, not commissions. The adviser can charge customers different fees for similar services without violating the Uniform Securities Act.

Dan is the owner of a mutual fund that returned him a before-tax return of 15% last year. Inflation is running at an annual rate of 3%, and Dan is in a 27% marginal income tax bracket. What has been Dan's approximate inflation-adjusted after-tax return on the fund over the course of the last year (rounded to the nearest 2 decimal points)? A) 12.00% B) 8.76% C) 7.95% D) 10.95%

C Explanation First, compute Dan's after-tax rate of return of 10.95% as follows: .15 × (1 − .27), or .73 = .1095. Then, compute Dan's inflation-adjusted, or real, rate of return by subtracting the 3% inflation rate from his 10.95% after-tax return. 7.95%

All of the following permit investments into various securities, such as stocks, bonds, and mutual funds EXCEPT A) a traditional IRA. B) a Roth IRA. C) an FSA. D) an HSA.

C Explanation Flexible spending accounts (FSAs) allow deductions from an employee's paycheck. That money is held by the company and is used to pay allowable claims by the employee. A health savings account (HSA) permits the employee to invest in a wide variety of securities. IRAs, traditional and Roth, have always permitted investment flexibility.

The general rules dealing with a broker-dealer extending credit for a customer to purchase securities are found in Regulation T of the Federal Reserve Board. However, Regulation T does NOT address A) loan value of securities B) initial margin requirements C) maintenance margin D) mixed margin accounts

C Explanation Maintenance margin levels are set by the SROs, such as FINRA. They are currently 25% for long accounts and 30% for short accounts (you will not have to calculate these).

A broker-dealer makes a market in XYZ stock and places large orders for it on the open market either at or slightly above its current price with the aim of stabilizing the price. This unethical practice is best described as A) front running B) straddling C) pegging D) matched orders

C Explanation Pegging involves entering buy orders for the purpose of supporting a stock price (i.e., to keep it from falling). This is a form of market manipulation and is illegal. Front running involves a representative or firm entering orders ahead of client orders. Straddles are an option position that combines a put and a call on the same stock; there is nothing improper with that strategy. Matched orders involve buying and selling a stock from one hand to the other to create the false appearance of trading volume and is another form of market manipulation.

One element of the formula used to compute the Sharpe ratio is A) beta B) net present value C) standard deviation D) alpha

C Explanation Standard deviation is used as the denominator in the formula used to compute the Sharpe ratio, a risk-adjusted rate of return. Standard deviation generally reflects the variability of portfolios that are not well diversified, while beta generally reflects the volatility of portfolios that are well diversified. Net present value measures the theoretical intrinsic value of an investment having uneven cash flows.

What new benefit did the TCJA of 2017 bring to 529 plans effective 2018? A) Tax-deductible contributions of up to $10,000 per year to pay for K-12 tuition B) Withdrawals may be made for qualified expenses at certain foreign educational institutions. C) Qualified withdrawals of up to $10,000 per year to pay for K-12 tuition D) Qualified withdrawals of up to $10,000 per year to pay for K-12 expenses

C Explanation The big change was the ability to use a 529 plan for K-12 expenses. However, the only expense that qualifies is tuition and there is a maximum limit of $10,000 per year. No contribution to any 529 is tax deductible. The use of the 529 for foreign educational institutions pre-dates the TCJA of 2017.

In general, in a defined benefit plan, the pension to be received upon retirement is based on the number of years of service and the individual's A) agreed salary. B) current salary. C) final salary. D) life expectancy.

C Explanation The final salary at retirement and the length of service at the employer are most commonly used to calculate the total benefit to be paid to the employee in the defined benefit pension plan.

A "margin account" is a type of brokerage account in which the broker-dealer lends the investor cash to purchase securities using marginable securities in the account as collateral. Which of the account documents authorizes the use of those securities as collateral for that loan? A) The loan consent agreement B) The credit agreement C) The hypothecation agreement D) The secured agreement

C Explanation The hypothecation agreement permits the broker-dealer to accept the client's margin securities as collateral for the loan that the BD makes to the investor. It also permits the BD to re-hypothecate those securities as collateral for a loan that it takes out to provide the money for the loan it makes to the investor. In simple terms, there are two loans taking place under the hypothecation agreement: The loan from the BD to the client with the client's securities used as collateral. The loan from a bank to the BD with the client's securities used as collateral for the BD's loan.

XYZ, Inc. is a C corporation in the 21% federal income tax bracket. Which of the following investments offers the company the highest after-tax return? A) Municipal bond with a 5% coupon rate B) REIT paying a 6.5% dividend C) ABCD, Inc. preferred stock paying a 6% dividend D) Corporate bond with a 6.75% coupon

C Explanation The key to this answer is that corporations have a 50% dividend exclusion on dividends received from other companies. The math looks like this: Only half of the 6% dividend is taxable. That means 3% per year is tax free and the other 3% is subject to tax at the 21% rate. So, we have 3% + 79% of the taxable 3% = 3% + 2.37% = 5.37% after-tax return. The municipal bond is not taxed, but that only produces 5% after tax. The corporate bond is subject to 21% tax so the corporation gets to retain the other 79%. That computes to 6.75 x 79% = 5.33%, just a bit less than the preferred stock. In most cases, dividends paid to corporations by REITs are fully taxable. That makes the after-tax return on the 6.5% dividend only 5.14%.

Which of the following statements are NOT true? The kiddie tax applies to any income received by a child under the age of 19. IRAs have advantages over other estate assets when left to charity. Simple trusts have to distribute income annually. For U.S. citizens, there is an unlimited marital estate tax deduction. A) I, II, III, and IV B) I, II, and III C) I and II D) II, III, and IV

C Explanation The kiddie tax applies to unearned income only such as that received in an UTMA account. Leaving IRA assets to a charity offers the same estate tax benefits as any other asset. Simple trusts must distribute income annually and there is an unlimited marital estate tax deduction between spouses who are U.S. citizens.

In order to compute an investor's real rate of return on a common stock holding, all of the following are necessary EXCEPT A) appreciation B) dividends C) marginal tax bracket D) inflation rate

C Explanation The real rate of return is another term for inflation-adjusted return. It is the total return, which is appreciation plus income adjusted for the inflation rate as expressed by the CPI. Tax bracket is necessary to compute after-tax return.

Which of the following takes place on the New York Stock Exchange? A) Buying and selling stocks on the over-the-counter (OTC) market B) Buying and selling of Nasdaq stocks C) Buying and selling stocks on the secondary market D) Buying and selling stocks on the primary market

C Explanation The secondary market is the market in which securities are traded after they are issued to the public. The secondary market takes place on exchanges, such as the New York Stock Exchange (NYSE), and on the over-the-counter (OTC) market. The OTC market is the market for securities that are not traded on an exchange.

Construction of an investment policy statement (IPS) requires identifying the client's objectives and constraints. Which of the following would not be in the list of constraints? A) Liquidity B) Taxes C) Risk tolerance D) Time horizon

C Explanation When constructing an investment policy statement (IPS) risk tolerance is an objective, not a constraint. Time horizon, taxes, and liquidity are all constraints. An easy way to remember the five constraints is TTLLU (time horizon, taxes, liquidity, laws, unique).

During the previous fiscal year, The Kaplan Family Trust received $24,000 in dividends and $35,000 in interest from corporate bonds. Securities transactions during the year resulted in long-term capital gains of $48,000, $20,000 of which were reinvested in the corpus. The DNI for the Kaplan Family Trust is A) $79,000 B) $107,000 C) $11,000 D) $87,000

D Explanation Distributable Net Income (DNI) is dividends and interest plus capital gains that have not been reinvested back into the trust. In this case, $24,000 + $35,000 + $28,000 = $87,000.

Which of the following statements about bid and asked prices are TRUE? The bid price is the price a dealer is willing to pay to buy a security. The asked price is the price a dealer is willing to accept to sell a security. The bid price for a security is higher than the asked price for the security. A) I and III B) II and III C) I, II, and III D) I and II

D Explanation The bid price is the price at which a dealer will buy a security, and the asked price is the price at which a dealer will sell. A dealer will always bid a lower price to buy a stock than to sell it.

Which of these is an advantage of using a Coverdell ESA rather than a 529 plan to fund a child's future education? A) The Coverdell allows for transfer of beneficiary. B) Contributions to the Coverdell are eligible for the annual gift tax exclusion. C) The Coverdell has greater tax advantages. D) The Coverdell offers greater investment flexibility.

D Explanation A Coverdell ESA works similar to a self-directed IRA where stocks, bond, mutual funds, ETFs, and other investment vehicles are options. With a 529 plan, the donor is limited to whatever is available in the state plan chosen. Tax advantages might be better for the 529 plan because many states allow a portion of the contribution to be taken as a deduction or credit against state income taxes. Both allow for transfer to a new beneficiary as long as that individual is a member of the original beneficiary's family. In both cases, whatever is contributed to the program is treated as a completed gift and is eligible for the annual gift tax exclusion.

Which of the following statements regarding IRAs are CORRECT? One may have both a Roth IRA and a traditional IRA, contributing the maximum to each one. One may have both a Roth IRA and a Roth 401(k) contributing the maximum to each one. Both traditional IRAs and Roth 401(k) plans have RMDs at age 72. If one is a participant in a Roth 401(k) plan, the earnings limits are waived for opening a Roth IRA. A) III and IV B) I and IV C) I and II D) II and III

D Explanation A Roth IRA and Roth 401(k) are 2 separate items, and maximum allowable contributions may be made to both. This is unlike the IRAs, where one can maintain both but the total contribution is the annual limit (currently $6,000 with a $1,000 catch-up). One of the things about a Roth 401(k) that is different from the Roth IRA is that RMDs must start at the same time as with traditional IRAs. Although one may participate in a Roth 401(k) without regard to AGI limits, that is not so with the Roth IRA.

An investor wishing to buy US Treasury bonds receives a quote from the dealer of 98.16. This represents A) a discount B) an indication of falling interest rates C) the offer price D) the bid price

D Explanation A dealer's quotes consist of the bid and the offer (ask). The bid price is what the dealer will pay a customer to purchase a security, and the offer is the dealer's selling price. In this case, the client wishes to purchase bonds, so the 98.16 represents the price the dealer is asking for them. Yes, the quote is a discount, but the better answer for this question is offer.

Tim earns $30,000 at his employment and is not offered a pension plan. His spouse is not currently employed. What is the best way to set up an IRA to give maximum retirement benefits? A) Set up separate accounts totaling $12,000. B) Set up one joint account for $12,000. C) Set up one IRA for $6,000 or 100% of earned income, whichever is less. D) Set up separate accounts for $6,000 each.

D Explanation A one-worker couple can open a spousal IRA. This type of arrangement allows the contribution of a total of $12,000 to the two accounts and no more than $6,000 in either account. Selecting separate accounts totaling $12,000 could imply that one account could exceed $6,000 while the other would be less. IRAs are always individual accounts. The spousal IRA allows contributions on behalf of a nonworking spouse.

One of your new clients has only been working for 3 years but is already interested in retirement planning. In order to be fully eligible for Social Security, the client must A) have minimum credited earnings of at least $20,000 per year. B) have at least 40 years of employment. C) be at least age 62. D) have a minimum of 40 covered quarters of employment.

D Explanation Current Social Security requirements are a minimum of 40 covered quarters of employment (10 years). A covered quarter is a calendar quarter during which the worker earned a minimum amount ($1,300 in 2017) which is indexed and, therefore, would never be tested. Reduced retirement benefits may begin as early as age 62, but disability payments can begin much, much earlier, as long as there have been 40 covered quarters. There is no minimum annual earnings limit.

On June 20, 2016, an investor in the 30% marginal federal tax bracket acquired a growth stock paying no dividend for $10 per share. On June 22, 2017, the investor sold the stock for $20 per share. Presuming capital gains rates are 15%, the investor's after-tax rate of return is closest to A) 200% B) 100% C) 70% D) 85%

D Explanation Although the stock grew at a 100% rate of return (by doubling), the investor must pay capital gains tax on the investment at 15%, and the investor realizes an after-tax rate of return of approximately 85%. Because the investor held the stock for more than 1 year, the sale is taxed at a favorable capital gains rate rather than at the investor's ordinary income tax rate.

In the administration of a qualified retirement plan, which of the following individuals is considered to be a fiduciary? A) The marketing director of the plan sponsor B) A CPA who prepares the plan's Form 5500 for an annual fee C) A highly compensated employee who participates in the plan D) A financial planner acting as a trustee over the plan assets

D Explanation An individual or business entity is considered a fiduciary under ERISA if that person renders investment advice or services to the plan for direct or indirect compensation.

An investor is reviewing his portfolio. To compute the real rate of return on an investment, it would be necessary to know all of the following EXCEPT A) the gain (or loss) recognized on the asset B) the rate of inflation C) the income received from holding the asset D) the tax bracket of the investor

D Explanation An investor's real rate of return is computed by dividing the total return received by the cost and then subtracting the inflation rate. If there is no realized gain, it is simply the nominal return based on the income minus the inflation rate. The investor's tax bracket is needed to compute after-tax returns. Return DIVIDED cost MINUS inflation

Terry Bolton opens a UTMA for each of his sons, Josh, age 12, and Drake, age 14. Under current tax regulations (2020 and beyond), after deductions and exemptions, how will the income in the UTMAs be taxed? Josh's income is taxed at his tax rate. Drake's income is taxed at his tax rate. Josh's income in excess of $2,200 is taxed at Terry's marginal tax rate. Drake's income in excess of $2,200 is taxed at Terry's marginal tax rate. A) I and IV B) II and III C) I and II D) III and IV

D Explanation Because the income on the UTMAs is not considered to be earned income, the kiddie tax rules apply. Currently (2020 and beyond, but indexed), children younger than 19 having such income in excess of $2,200 are subject to tax at the parent's marginal tax rate. That means if the parent is in the 32% income tax bracket, the children's excess income will be taxed at 32%.

Dan is the owner of a mutual fund that returned him a before-tax return of 15% last year. Inflation is running at an annual rate of 3%, and Dan is in a 27% marginal income tax bracket. What has been Dan's approximate inflation-adjusted after-tax return on the fund over the course of the last year (rounded to the nearest 2 decimal points)? A) 12.00% B) 8.76% C) 10.95% D) 7.95%

D Explanation First, compute Dan's after-tax rate of return of 10.95% as follows: .15 × (1 − .27), or .73 = .1095. Then, compute Dan's inflation-adjusted, or real, rate of return by subtracting the 3% inflation rate from his 10.95% after-tax return.

Under Keogh plan provisions, a full-time employee is defined as one working at least how many hours per year? A) 500 B) 2,000 C) 100 D) 1,000

D Explanation Full-time employment is defined as 1,000 hours or more per year, regardless of the number of days, weeks, or months worked.

Which of the following statements about capital gains are TRUE? The minimum holding period required to qualify for long-term capital gains treatment is 1 day longer than 12 months. The highest federal income tax rate on long-term capital gains is less than the highest federal income tax rate on ordinary income. If an investor holds stock for 12 months or less and has no other transactions, any gain on the sale of the stock is taxed at the same rate as ordinary income. A) II and III B) I and III C) I and II D) I, II, and III

D Explanation If an investor holds stock for more than 12 months and sells it for a gain, the gain will be treated as a long-term capital gain. The advantage of long-term capital gains is that the maximum tax rate on long-term capital gains is lower than the maximum rate on ordinary income. If an investor holds stock for 12 months or less, though, any gain will be considered a short-term capital gain and will be taxed at the same rate as ordinary income.

Each of the following individuals is eligible to participate in a Keogh plan EXCEPT A) an engineer employed by a corporation who earns $5,000 making public speeches in her spare time B) a self-employed doctor in private practice C) a securities analyst employed by a major research organization who makes $2,000 giving lectures in his spare time D) an executive of a corporation who receives $5,000 in stock options from his company

D Explanation Individuals with income from self-employment may participate in Keogh plans. Stock options, capital gains, dividends, and interest are not considered income earned from self-employment.

A company with 20 million shares outstanding paid $36 million in dividends. If the current market value of the company's shares is $36, the current yield is A) not determinable from the information given B) 2% C) 10% D) 5%

D Explanation The current yield formula is annual dividends per share divided by current market price. The dividends per share are $36 million ÷ 20 million shares = $1.80 per share. Current yield is $1.80 ÷ $36.00 = 5%.

An individual purchased a variable life insurance policy 10 years ago with a guaranteed death benefit of $100,000. The annual premium for this policy was $2,000 per year. The individual dies and, due to outstanding performance of the separate account, leaves a death benefit to the beneficiary of $121,000. What are the income tax consequences to that beneficiary? A) There is a long-term capital gain of $1,000. B) Ordinary income tax is due on $21,000. C) Ordinary income tax is due on the $1,000 that exceeds the original cost. D) No tax is due.

D Explanation One of the nice things about life insurance proceeds is that even when the death benefit is increased due to separate account performance, it is still free of income tax.

The donor to a 529 plan has decided to move the existing plan to one offered by another state. Which of the following statements is NOT true? A) If there is a distribution of the assets, the rollover must be completed within 60 days. B) Unless a change of beneficiary is involved, only one rollover is permitted in a 12 month period. C) Even though these plans are generally under state control, the rollover rules are federal law. D) This may be done, but only if the entire account is rolled over.

D Explanation Partial rollovers are permitted.

A customer who is changing jobs has how many days to roll over a lump-sum distribution from a qualified pension plan into an IRA? A) 30 days B) 90 days C) 15 days D) 60 days

D Explanation Rollovers must be completed within 60 days of the distribution date to avoid unfavorable tax consequences.

Which of the following entities would issue a Schedule K-1? A) Sole proprietorship B) C corporation C) REIT D) Limited partnership

D Explanation Schedule K-1s are issued to owners of partnerships (limited or general), LLCs with more than one member, and S corporations. Sole proprietors use a Schedule C, C corporations report dividends and/or interest paid on a Form 1099, and the same is true for distributions from a REIT.

Which of the following investments could be found in an UTMA but not an UGMA? A) Bonds B) Sector mutual fund C) Preferred stock D) Real estate

D Explanation The Uniform Transfer to Minors Act (UTMA) allows virtually any kind of asset, including real estate, to be transferred to a minor. UGMA accounts, on the other hand, are limited to gifts of cash, securities (such as stocks, bonds, or mutual funds), and insurance policies.

Which of the following investments is the most liquid? A) Common stock in a small oil drilling corporation that is quoted on the OTC Link B) Municipal revenue bond issued by a township C) Oil drilling limited partnership interest D) Long-term municipal bond fund

D Explanation The long-term municipal bond fund is the most liquid because it is a mutual fund (a redeemable security), and the investor is assured of a buyer that will exchange money for the redeemed fund shares within 7 days of the redemption request. Municipal bonds of a township, especially those that are from extremely small issuers, may have thin trading markets where sellers have difficulty finding willing buyers. There is not an active secondary market for reselling interests in limited partnerships. Stock of a small corporation that trades on the OTC Link (formerly known as the "Pink Sheets") may also have a thin trading market.

When an investor's original value is subtracted from the ending value, and then has the income received over that time period added to it, which is then divided by the original cost, the result is A) annualized return B) internal rate of return C) expected return D) holding period return

D Explanation This is the method of computing holding period return. OV - EV + Income DIVIDED by OC

An investor purchases 100 shares of RIF common stock. In the year following the purchase, the RIF shares appreciated by 12% and paid a 2% dividend. If inflation, as measured by the CPI, was at a 4% rate, the investor's total return on the RIF shares is closest to A) 8% B) 12% C) 10% D) 14%

D Explanation This question is asking for the total return, which is 14% (12% appreciation + 2% dividend). Had the question asked for the inflation-adjusted return, (which it doesn't), that is 14% minus the 4% CPI.

A broker-dealer acting as a principal in a trade would A) add a markup to the bid price when offering shares to a client B) must always disclose the amount of markup on a client's confirmation statement C) must disclose to clients the amount of earnings he made on principal transactions in excess of the amount he would have made had he charged a commission D) add a markup to the offering price when selling shares to a client

D Explanation When selling a security to a public customer, the broker-dealer adds his markup to the ask price (offer price), not the bid price. A broker does not add a markup to the bid price when buying shares from a client; the broker-dealer would mark down the bid price. Unlike commissions, which are always disclosed on the trade confirmations, only for certain categories of securities is the markup or markdown shown.


Set pelajaran terkait

Monopolistic Competition & Oligopoly

View Set

Chapter 11 - JAVA PROGRAMMING Inheritance and Polymorphism

View Set

Chapter 14: Small Business Finance: Using Equity, Debt, and Gifts

View Set

35Qw/exp Module 8: HIV Infection & Aids

View Set

Introduction to Public Health Final Exam

View Set

Bless Me Ultima Review Questions

View Set

Ch. 17 Go West Young Man! Westward Expansion, 1840-1900

View Set

Chapter 11: Consideration and a Promissionary Estoppel - DONE

View Set